Как найти частоту приемника

как найти частоту радиоприемника если нам известна длинна волны?

Максим Никулин



Мастер

(2291),
закрыт



3 года назад

как длину через частоту знаю а вот частоту через длину…

Павел Пацевич

Мудрец

(12479)


3 года назад

А что тут сложного? Длина= скорость света/ частоту

Максим НикулинМастер (2291)

3 года назад

Да не длину надо найти а частоту. вот допустим длинна волны передатчика 20м, на какой он частоте работает?

Павел Пацевич
Мудрец
(12479)
Соответственно частота=скорость света / длину волны. Скорость света примерно 300000000 м/с, длина волны если 20м, то получаем 15 000000 колебаний в секунду, которые называются Герцами, стало быть 15 мегагерц.

Содержание

  1. Длина, скорость и частота электромагнитной волны.
  2. Как узнать частоту ИК пульта ДУ?
  3. Как найти частоту приемника
  4. О колебаниях и волнах
  5. Период, частота, амплитуда колебаний
  6. Что такое радиоволны?
  7. Радиовещание. Радиовещательные диапазоны волн
  8. Радиопередача
  9. Распространение радиоволн
  10. Как работает радиоприемник?
  11. Детектор и детектирование радиосигнала
  12. Головной телефон и его устройство
  13. Практическая работа
  14. Принципиальная электрическая схема детекторного приемника

Длина, скорость и частота электромагнитной волны.

Онлайн калькулятор перевода длины волны в частоту для широкого диапазона частот, включая радиоволны, микроволны, инфракрасное излучение, видимый свет, ультрафи- олетовое излучение, рентгеновские и гамма лучи.

Электромагнитные колебания — это взаимосвязанные колебания электрического и магнитного полей, проявляющиеся в периодическом изменении напряжённости (E) и индукции (B) поля в электроцепи или пространстве. Эти поля перпендикулярны друг другу в направлении движения волны (Рис.1) и, в зависимости от частоты, представляют собой: радиоволны, микроволны, инфракрасное излучение, видимый свет, ультрафиолетовое излучение, рентгеновские либо гамма-лучи.


Рис.1

Длина волны, обозначаемая буквой λ и измеряемая в метрах — это расстояние между двумя ближайшими друг к другу точками в пространстве, в которых колебания происходят в одинаковой фазе. Другими словами, это расстояние, на котором фаза электромагнитной волны вдоль направления распространения меняется на 2π.

Время, за которое волна успевает преодолеть это расстояние (λ), т. е. интервал времени, за который периодический колебательный процесс повторяется, называется периодом колебаний, обозначается буквой (тау) или Т и измеряется в метрах.

Частота электромагнитных колебаний связана с периодом простейшим соотношением:
f (Гц) = 1 / T (сек) .

Скорость распространения электромагнитных волн в вакууме (v) равна скорости света и составляет величину: v = С = 299792458 м/сек .
В среде эта скорость уменьшается: v = С / n , где n > 1 — это показатель преломления среды.
Абсолютный показатель преломления любого газа (в том числе воздуха) при обычных условиях мало чем отличается от единицы, поэтому с достаточной точностью его можно не учитывать в условиях распространения электромагнитных волн в воздушном пространстве.

Соотношение, связывающее длину волны со скоростью распространения в общем случае, выглядит следующим образом:
λ (м) = v (м/сек) *Т (сек) = v (м/сек) / f (Гц) .

И окончательно для воздушной среды:

λ (м) = 299792458 *Т (сек) = 299792458 / f (Гц) .

Прежде чем перейти к калькуляторам, давайте рассмотрим шкалу частот и длин волн непрерывного диапазона электромагнитных волн, которая традиционно разбита на ряд поддиапазонов. Соседние диапазоны могут немного перекрываться.

Диапазон Полоса частот Длина волны
Сверхдлинные радиоволны 3. 30 кГц 100000. 10000 м
Длинные радиоволны 30. 300 кГц 10000. 1000 м
Средние радиоволны 300. 3000 кГц 1000. 100 м
Короткие радиоволны 3. 30 МГц 100. 10 м
Метровый радиодиапазон 30. 300 МГц 10. 1 м
Дециметровый радиодиапазон 300. 3000 МГц 1. 0,1 м
Сантиметровый СВЧ диапазон 3. 30 ГГц 10. 1 см
Микроволновый СВЧ диапазон 30. 300 ГГц 1. 0,1 см
Инфракрасное излучение 0,3. 405 ТГц 1000. 0,74 мкм
Красный цвет 405. 480 ТГц 740. 625 нм
Оранжевый цвет 480. 510 ТГц 625. 590 нм
Жёлтый цвет 510. 530 ТГц 590. 565 нм
Зелёный цвет 530. 600 ТГц 565. 500 нм
Голубой цвет 600. 620 ТГц 500. 485 нм
Синий цвет 620. 680 ТГц 485. 440 нм
Фиолетовый цвет 680. 790 ТГц 440. 380 нм
Ультрафиолетовое излучение 480. 30000 ТГц 400. 10 нм
Рентгеновское излучение 30000. 3000000 ТГц 10. 0,1 нм
Гамма излучение 3000000. 30000000 ТГц 0,1. 0,01 нм

А теперь можно переходить к калькуляторам.

КАЛЬКУЛЯТОР РАСЧЁТА ДЛИНЫ ВОЛНЫ ПО ЧАСТОТЕ

КАЛЬКУЛЯТОР РАСЧЁТА ЧАСТОТЫ ПО ДЛИНЕ ВОЛНЫ

В радиочастотной практике имеет распространение величина Kp, называемая коэффициентом укорочения. Однако здесь существует некоторая путаница. Одни источники интерпретируют эту величину, как отношение длины волны в среде к длине волны в вакууме, т. е. численно равной Kp = 1/n, где n — это, как мы помним, показатель преломления среды. Другие, наоборот — как отношение длины волны в вакууме к длине волны в среде, т. е. Kp = n.
Поэтому надо иметь в виду — если Kp > 1, то значение показателя преломления среды, которое следует подставлять в калькулятор n = Kp, а если Kp < 1, то n = 1/Kp.

Источник

Как узнать частоту ИК пульта ДУ?

Есть телевизор Sony, есть пульт от него. Как узнать частоту работы пульта, чтобы подобрать ИК приемник под него?

Смотрел документацию по телевизору, отдельно по пульту, юзал Google, информации 0. Только примерные данные на уровне «может быть» и «скорее всего».

Как точно узнать его частоту, чтобы подобрать соответствующий приемник?

Заранее спасибо за ответы.

  • Вопрос задан более трёх лет назад
  • 50869 просмотров

Оценить 1 комментарий

Давайте для начала прочтем заголовок темы «Как узнать частоту ИК пульта ДУ?».
Именно такой я загуглил запрос.
И что Вы отвечаете?
-На любом приемнике будет работать.
-Пульты все одинаковые.
-У меня по этой схеме все работает.

Хорошо, усложняем задачу.
У меня есть тагер (оружие для лазертага), который «стреляет» ИК сигналами.
VS1838 принимает сигналы, но с искажениями, так как частота не совпадает.
Итак вопрос: Как узнать частоту ИК сигнала?

Источник

Как найти частоту приемника

Слово «радио» происходит от латинского radiare — излучать или испускать лучи. Радиовещательная станция, например, подобно Солнцу излучает радиоволны во все стороны по радиусам. Лишь некоторые радиостанции специального назначения излучают радиоволны в каком — то одном направлении. Если бы вы пришли на территорию радиовещательной станции, то прежде всего увидели бы вертикальную ажурную металлическую мачту или провода, поднятые высоко над землей. Это — антенна. Рядом или неподалеку — здание, где находится передатчик, вырабатывающий электрические колебания высокой частоты, которые антенна преобразует в энергию радиоволн. К передатчику от радиостудии, а она может находиться далеко от передатчика, идет подземный кабель — хорошо изолированные провода в прочной оболочке. В студии установлен микрофон. Не только голос диктора, разговор людей и звуки музыки, но и шепот, шорохи микрофон мгновенно превращает в электрические колебания звуковой частоты, которые по кабелю поступают к передатчику. Скольким еще преобразованиям подвергается переменный ток звуковой частоты, прежде чем приемник превратит его снова в звуки. Приемник будет первым вашим практическим шагом к познанию радиотехники. А чтобы этот шаг был уверенным, надо разобраться в сущности тех физических явлений, которые лежат в основе техники радиопередачи и радиоприема, поговорить о природе звука, переменном токе и его свойствах.

О колебаниях и волнах

Вокруг нас все время рождаются и затухают колебательные явления. Колеблется ветка, с которой слетела птица. Колеблются маятники часов, качели. Под действием ветра колеблются деревья, провода, подвешенные на столбах, колеблется вода в озерах и морях. Вот вы бросили на гладкую поверхность озера камень, и от него побежали волны. Что произошло? Частицы воды в месте удара камня вдавились, вытеснив соседние частицы, и на поверхности воды образовался кольцеобразный горб. Затем в месте падения камня, вода поднялась вверх, но уже выше прежнего уровня — за первым горбом появился второй, а между ними — впадина. Далее частицы воды продолжают перемещаться попеременно вверх и вниз — колеблются, увлекая за собой все больше и больше соседних частиц воды. Образуются волны, расходящиеся от места своего возникновения концентрическими кругами. Подчеркиваю: частицы воды только колеблются, но не движутся вместе с волнами. В этом нетрудно убедиться, бросив на колеблющуюся поверхность воды щепку. Если нет ветра или течения воды, щепка будет лишь опускаться и подниматься над уровнем воды, не перемещаясь вместе с волнами. Водяные волны могут быть большими, т.е. сильными или маленькими — слабыми. Сильными мы называем такие волны, которые имеют большой размах колебаний, как говорят, большие амплитуды колебаний. Слабые волны имеют малые горбы — небольшую амплитуду. Чем больше амплитуды возникших волн, тем большую энергию они несут в себе. Энергия волн, возникших от брошенного камня, относительно невелика, однако она может заставить колебаться камыш и траву, растущие в озере. Но мы знаем, какие большие разрушения берега могут производить морские волны, обладающие большими амплитудами и, следовательно, большой энергией. Эти разрушения осуществляются именно той энергией, которую волны непрерывно отдают берегу. Волны могут быть частыми или редкими. Чем меньше расстояние между гребнями бегущих волн, тем короче каждая взятая в отдельности волна. Чем больше расстояние между волнами, тем длиннее волна. Длиной волны на воде мы называем расстояние между двумя соседними бегущими гребнями или впадинами. По мере удаления волн от места возникновения их амплитуды постепенно уменьшаются, затухают, но длина волн остается неизменной. Волны на воде можно также создавать, например, погружая в воду палку и ритмично, в такт с колебаниями воды, опуская и поднимая ее. И в этом случае волны будут затухающими. Но существовать они будут лишь до тех пор, пока мы не прекратим возмущать поверхность воды. А как возникают колебания обычных качелей? Это вы хорошо знаете: их надо подтолкнуть, вот они и будут качаться из стороны в сторону. Чем сильнее толчок, тем больше амплитуды колебаний. Эти колебания будут затухать, если не поддерживать их дополнительными толчками. Такие и многие другие подобные механические колебания мы видим. В природе же больше невидимых колебаний, которые мы слышим, ощущаем в виде звука. Не всегда, например, можно заметить колебания струны музыкального инструмента, но мы слышим, как она звучит. При порывах ветра в трубе возникает звук. Его создают колебательные движения воздуха в трубе, которые мы не видим. Звучат камертон, стакан, ложка, тарелка, ученическое перо, лист бумаги — они тоже колеблются. Да, мы живем в мире звуков, потому что многие окружающие нас тела, колеблясь, звучат. Как возникают звуковые волны в воздухе? Воздух состоит из невидимых глазам частиц. При ветре они могут переноситься на большие расстояния. Но они, кроме того, могут и колебаться. Например, если в воздухе сделать резкое движение палкой, то мы почувствуем легкий порыв ветра и одновременно услышим слабый звук. Звук этот — результат колебаний частиц воздуха, возбужденных колебаниями палки. Проведем такой опыт. Оттяним струну, например, гитары, а потом отпустим ее. Струна начнет дрожать — колебаться около своего первоначального положения покоя. Достаточно сильные колебания струны заметны на глаз. Слабые колебания струны можно только почувствовать как легкое щекотание, если прикоснуться к ней пальцем. Пока струна колеблется, мы слышим звук. Как только струна успокоится, звук затихнет. Рождение звука здесь — результат сгущения и разрежения частиц воздуха. Колеблясь из стороны в сторону, струна теснит, как бы прессует перед собой частицы воздуха, образуя в некотором его объеме области повышенного давления, а сзади, наоборот, области пониженного давления. Это и есть звуковые волны. Распространяясь в воздухе со скоростью около 340 м/с, они несут в себе некоторый запас энергии. В тот момент, когда до ухода доходит область повышенного давления звуковой волны, она надавливает на барабанную перепонку, несколько прогибая ее внутрь. Когда же до уха доходит разреженная область звуковой волны, барабанная перепонка выгибается несколько наружу. Барабанная перепонка все время колеблется в такт с чередующимися областями повышенного и пониженного давления воздуха. Эти колебания передаются по слуховому нерву в мозг, и мы воспринимаем их как звук. Чем больше амплитуды звуковых волн, тем больше энергии несут они в себе, тем громче воспринимаемый нами звук. Звуковые волны, как и водяные или электрические колебания, изображают волнистой линией — синусоидой. Ее горбы соответствуют областям повышенного давления, а впадины — областям пониженного давления воздуха. Область повышенного давления и следующая за нею область пониженного давления образуют звуковую волну. Мы живем и в мире электромагнитных колебаний, излучаемых электрическими приборами и всеми проводами, в которых течет переменный ток, огромным числом антенн радиостанций, атмосферными электрическими разрядами, недрами Земли и бесконечным Космосом. Только с помощью приборов, созданных человеком, они могут быть обнаружены и зафиксированы.

Период, частота, амплитуда колебаний

Важнейшим параметром, характеризующим механические, звуковые, электрические, электромагнитные и все другие виды колебаний является период — время, в течение которого совершается одно полное колебание. Если, например, маятник часов — ходиков делает за 1 с два полных колебания, период каждого колебания равен 0,5 с. Период колебаний больших качелей около 2 с, а период колебаний струны может составлять от десятых до десятитысячных долей секунды. Другим параметром, характеризующим колебания, является частота (от слова «часто») — число, показывающее, сколько полных колебаний в секунду совершают маятник часов, звучащее тело, ток в проводнике и т. п. Частоту колебаний оценивают единицей, носящей название Герц (сокращенно пишут Гц): 1 Гц — это одно колебание в секунду. Если, например, звучащая струна совершает 440 полных колебаний в 1 с (при этом она создает тон «ля» третьей октавы), говорят, что частота ее колебаний 440 Гц. Частота переменного тока электроосветительной сети 50 Гц. При этом токе электроны в проводах сети в течение секунды текут попеременно 50 раз в одном направлении и столько же раз в обратном, т. е. совершают за 1 с 50 полных колебаний. Более крупные единицы частоты — килогерц (пишут кГц), равный 1000 Гц и мегагерц (пишут МГц), равный 1000 кГц или 1000000 Гц. По частоте колебаний звучащего тела можно судить о тоне или высоте звука. Чем больше частота, тем выше тон звука, и наоборот, чем меньше частота, тем ниже тон звука. Наше ухо способно реагировать на сравнительно небольшую полосу (участок) частот звуковых колебаний — примерно от 20 Гц до 20 кГц. Тем не менее эта полоса частот вмещает всю обширнейшую гамму звуков, создаваемых голосом человека, симфоническим оркестром: от очень низких тонов, похожих на звук жужжания жука, до еле уловимого высокого писка комара. Колебания частотой до 20 Гц, называемые инфразвуковыми, и свыше 20 кГц, называемые ультразвуковыми, мы не слышим.А если бы барабанная перепонка нашего уха оказалась способной реагировать и на ультразвуковые колебания, мы могли бы тогда услышать писк летучих мышей, голос дельфина. Дельфины издают и слышат ультразвуковые колебания с частотами до 180 кГц. Но, не путайте высоту, т. е. тон звука с его силой. Высота звука зависит не от амплитуды, а от частоты колебаний. Толстая и длинная струна музыкального инструмента, например, создает низкий тон звука, т. е. колеблется медленнее, чем тонкая и короткая

Рис. 1. Чем больше частота колебаний струны, тем короче звуковые волны и выше тон звука.

струна, создающая высокий, тон звука. Разобраться в этом вопросе вам поможет (рис. 1). В электро — и радиотехнике используют переменные токи частотой от нескольких герц до тысяч гигагерц. Антенны широковещательных радиостанций, например, питаются токами частотой примерно от 150 кГц до 100 МГц. Эти быстропеременные колебания, называемые колебаниями радиочастоты, и являются тем средством, с помощью которого осуществляется передача звуков на большие расстояния без проводов. Весь огромный диапазон переменных токов принято подразделять на несколько участков — поддиапазонов. Токи частотой от 20 Гц до 20 кГц, соответствующие колебаниям, воспринимаемым нами как звуки разной тональности, называют токами (или колебаниями), звуковой частоты, а токи частотой выше — 20 кГц — токами ультразвуковой частоты. Токи частотой от 100 кГц до 30 МГц называют токами высокой частоты, а токи частотой выше 30 МГц — токами ультравысокой и сверхвысокой частоты. Запомните эти границы и названия поддиапазонов частот переменных токов.

Что такое радиоволны?

Предположим, вы снимаете трубку телефонного аппарата, набираете или называете нужный номер. Вскоре вы слышите голос товарища, а он — ваш. Какие электрические явления происходят во время вашего телефонного разговора? Звуковые колебания воздуха, созданные вами, преобразуются микрофоном в электрические колебания звуковой частоты, которые по проводам передаются к аппаратуре вашего собеседника. Там, на другом конце линии, они с помощью излучателя телефона преобразуются в колебания воздуха, воспринимаемые вашим приятелем как звуки. В телефонии средством связи цепи являются провода, в радиовещании — радиоволны. «Сердцем» передатчика любой радиостанции является генератор — устройство, вырабатывающее колебания высокой, но строго постоянной для данной радиостанции частоты. Эти колебания радиочастоты, усиленные до необходимой мощности, поступают в антенну и возбуждают в окружающем ее пространстве электромагнитные колебания точно такой же частоты — радиоволны. Скорость удаления радиоволн от антенны радиостанции равна скорости света: 300000 км/с, что почти в миллион раз быстрее распространения звука в воздухе. Это значит, что если на Московской радиовещательной станции в некоторый момент времени включили передатчик, то ее радиоволны меньше чем за 1/30с дойдут до Владивостока, а звук за это время успеет распространиться всего лишь на 10 — 11м. Радиоволны распространяются не только в воздухе, но и там, где его нет, например в космическом пространстве. Этим они отличаются от звуковых волн, для которых совершенно необходим воздух или какая — либо другая плотная среда, например вода. Когда радиовещательная станция начинает свои передачи, диктор иногда сообщает, что данная радиостанция работает на волне такой — то длины. Волну, бегущую по поверхности воды, мы видим и при известной ловкости можем измерить ее длину. Длину же радиоволн можно измерить только с помощью специальных приборов или рассчитать математическим путем, если, конечно, известна частота тока, возбуждающего эти волны. Длина радиоволны — это расстояние, на которое распространяется энергия электромагнитного поля за период колебания тока в антенне радиостанции. Понимать это надо так. За время одного периода тока в антенне передатчика в пространстве вокруг нее возникает одна радиоволна. Чем выше частота тока, тем больше следующих друг за другом радиоволн излучается антенной в течение каждой секунды. Допустим, частота тока в антенне радиостанции составляет 1 МГц. Значит, период этого тока и возбужденного им электромагнитного поля равен одной миллионной доле секунды. За 1 с радиоволна проходит расстояние 300000 км, или 300000000 м. За одну миллионную долю секунды она пройдет расстояние в миллион раз меньше, т.е 300 000 000 : 1000 000. Следовательно длина волны данной радиостанции равна 300 м. Итак, длина волны радиостанции за висит от частоты тока в ее антенне: чем больше частота тока, тем короче волна, и наоборот, чем меньше частота тока, тем длиннее волна. Чтобы узнать длину волны радиостанции, надо скорость распространения радиоволн, выраженную в метрах, разделить на частоту тока в ее антенне. И наоборот, чтобы узнать частоту тока в антенне радиостанции, надо скорость распространения радиоволн разделить на длину волны этой радиостанции. Для перевода частоты тока передатчика в мегагерцах в длину волны в метрах и обратно удобно пользоваться такими формулами: ?(м) = ЗОО/f (МГц); f (МГц) = ЗОО/?, (м), где ? (греческая буква «лямбда») — длина волны; f — частота колебаний, 300 — скорость распространения радиоволн, выраженная в тысячах километров в секунду. Хочу вас предупредить: не путайте понятие о длине волны, на которой работает радиостанция, с дальностью ее действия, т. е. с расстоянием, на котором передачи этой станции могут быть приняты. Дальность действия радиостанции, правда, зависит от длины волны, но не отождествляется с нею. Так, передача станции, работающей на волне длиной в несколько десятков метров, может быть услышана на расстоянии в несколько тысяч километров, но не всегда слышна на более близких расстояниях. В то же время передача радиостанции, работающей на волне длиной в сотни и тысячи метров, часто не слышна на таких больших расстояниях, на которых слышны передачи коротковолновых станций. Итак, каждая радиовещательная станция работает на определенной, отведенной для нее частоте, называемой несущей. Длины волн различных радиостанций неодинаковы, но строго постоянны для каждой из них. Это и дает возможность принимать передачи каждой радиостанции в отдельности, а не все одновременно.

Радиовещание. Радиовещательные диапазоны волн

Весьма широкий участок радиоволн, отведенный для радиовещательных станций, условно подразделен на несколько диапазонов: длинноволновый (сокращенно ДВ), средневолновый (сокращенно СВ), коволновый (сокращенно KB), ультракоротковолновый (УКВ). В странах СНГ длинноволновый диапазон охватывает радиоволны длиной от 735,3 до 2000 м, что соответствует частотам 408-150 кГц; средневолновый — радиоволны длиной от 186,9 до 571,4 м (радиочастоты 1605 — 525 кГц); коротковолновый — радиоволны длиной от 24,8 до 75,5 (радиочастоты 12,1 — 3,95 МГц); ультракоротковолновый — радиоволны длиной от 4,11 до 4,56 м (радиочастоты 73 — 65,8 МГц). Радиоволны УКВ диапазона называют также метровыми волнами; вообще же ультракороткими волнами называют все волны короче 10 м. В этом диапазоне ведутся телевизионные передачи, работают связные радиостанции, оборудованные на автомашинах пожарной охраны, такси, медицинского обслуживания населения на дому и т. д. Радиочастоты коротковолновых вещательных станций неравномерно распределены по диапазону: больше всего их работает на волнах длиной около 25, 31, 41 и 50 м. Соответственно поэтому коротковолновый радиовещательный диапазон подразделяется на 25, 31, 41 и 50-метровый поддиапазоны. Согласно международному соглашению волна длиной 600 м (500 кГц) отведена для передачи сигналов бедствия кораблями в море — SOS. На этой волне работают все морские аварийные радиопередатчики, на эту волну настроены приемники спасательных станций и маяков.

Радиопередача

Если сложное техническое оснащение радиовещательной станции изобразить упрощенно в виде условных знаков и прямоугольников, то получится ее структурная схема в таком виде, как показано на рис. 2. Здесь пять основных приборов и устройств: студийный микрофон, усилитель звуковой частоты (3Ч), генератор колебаний радиочастоты (РЧ), усилитель мощности колебаний радиочастоты и антенна, излучающая электромагнитную энергию радиоволн. Пока студийный микрофон не включен, в антенне станции течет ток высокой (несущей), но строго постоянной частоты и амплитуды (см. левые части графиков на рис. 3). Антенна при этом излучает радиоволны неизменной длины и мощности. Но вот в студии включили микрофон, и люди, находящиеся за десятки, сотни и тысячи километров от радиостанции, услышали знакомый голос диктора.

Рис. 2 Структурная схема радиовещательной станции.

Рис. 3 При действии звука на микрофон ток высокой частоты в антенне передатчика изменяется по амплитуде.

Что же в это время происходит в передатчике радиостанции? Колебания звуковой частоты, созданные микрофоном и усиленные студийным усилителем 3Ч, подают на так называемый модулятор, входящий в усилитель мощности передатчика, и там, воздействуя на ток высокой частоты генератора, изменяют его амплитуду колебаний. От этого изменяется излучаемая антенной передатчика электромагнитная энергия (см. правые части графиков на рис. 3). Чем больше частота тока, поступающего из радиостудии в передатчик, тем с большей частотой изменяются амплитуды тока в антенне. Так звук, преобразованный микрофоном в электрические колебания звуковой частоты, получает «путевку» в эфир. Процесс изменения амплитуд высокочастотных колебаний под действием тока звуковой частоты называют амплитудной модуляцией (AM). Изменяемые же по амплитуде токи высокой частоты в антенне и излучаемые ею радиоволны носят название модулированных колебаний радиочастоты. Кроме амплитудной модуляции существует еще так называемая частотная модуляция (ЧМ). При таком виде модуляции изменяется частота, а амплитуда колебаний радиочастоты в антенне радиостанции остается неизменной. Частотную модуляцию применяют, например, для передачи звукового сопровождения в телевидении, в радиовещании на УКВ. В радиовещании на ДВ, СВ и KB используют только амплитудную модуляцию. Радиоволны не могут быть обнаружены ни одним органом наших чувств. Но если на их пути встречается проводник, они отдают ему часть своей энергии. На этом явлении и основан прием радиопередач. Улавливание энергии радиоволн приемником осуществляет антенна радиоприемника. Отдавая антенне часть электромагнитной энергии, радиоволны индуцируют в ней модулированные колебания радиочастоты. В приемнике имеют место процессы, обратные тем, которые происходят в студии и на передатчике радиостанции. Если там звук последовательно преобразуют сначала в электрические колебания звуковой частоты, а затем в модулированные колебания радиочастоты, то при радиоприеме решается обратная задача: модулированные колебания радиочастоты, возбужденные в антенне, приемник преобразует в электрические колебания звуковой частоты, а затем в звук. В простейшем приемнике, работающем только благодаря энергии, уловленной антенной, модулированные колебания радиочастоты преобразуются в колебания звуковой частоты детектором, а эти колебания в звук — головными телефонами. Но ведь антенну приемника пронизывают радиоволны множества радиостанций, возбуждая в ней модулированные колебания самых различных радиочастот. И если все эти радиосигналы преобразовать в звуки, то мы услышали бы сотни голосов людей, разговаривающих на разных языках. Вряд ли такой радиоприем нас устроил. Разумеется, интересно послушать передачи разных станций, но только, конечно, не все одновременно, а каждую в отдельности. А для этого из колебаний всех частот, возбуждающихся в антенне, надо выделить колебания с частотой той радиостанции, передачи которой хотим слушать. Эту задачу выполняет колебательный контур, являющийся обязательной частью как самого простого так и самого сложного радиовещательного приемника. Именно с помощью колебательного контура вы будете настраивать свой первый приемник на сигналы радиостанций разной длины волны.

Распространение радиоволн

Сейчас рассмотрим некоторые особенности распространения радиоволн. Дело в том, что радиоволны разных диапазонов обладают неодинаковыми свойствами, влияющими на дальность их распространения. Волны одной длины преодолевают большие расстояния, волны другой длины «теряются» за пределами горизонта. Бывает так, что радиосигнал превосходно слышен где — то по ту сторону Земли или в Космосе, но его невозможно обнаружить в нескольких десятках километров от радиостанции. Чем это объяснить? Что влияет на «дальнобойность» радиоволн разной длины? Земля и окружающая ее атмосфера. Земля — проводник тока, хотя и не такой хороший, как, скажем медные провода. Земная атмосфера состоит из трех слоев. Первый слой, верхняя граница которого кончается в 10 — 12 км от поверхности Земли, называется тропосферой. Над ней километров до 50 от поверхности Земли, второй слой — стратосфера. А выше, примерно до 400 км над Землей, простирается третий слой ионосфера (рис. 4). Ионосфера играет решающую роль в распространении радиоволн, особенно коротких.

Рис. 4 Распространение радиоволн.

Воздух в ионосфере сильно разрежен. Под действием солнечных излучений там из атомов газов выделяется много свободных электронов, в результате чего появляются положительные ионы. Происходит, как говорят, ионизация верхнего слоя атмосферы. Ионизированный слой способен поглощать радиоволны и искривлять их путь. В течение суток в зависимости от интенсивности солнечного излучения количество свободных электронов в ионизированном слое, его толщина и высота изменяются, а от этого изменяются и электрические свойства этого слоя. Антенны радиостанций излучают радиоволны вдоль поверхности Земли и вверх под различными углами к ней. Волны, идущие вдоль поверхности, называют земными или поверхностными, под различными углами — пространственными. При передаче сигналов ДВ станций используется главным образом энергия поверхностных волн, которые хорошо огибают поверхность Земли. Но Земля, являясь проводником, поглощает энергию радиоволн. Поэтому по мере удаления от ДВ станций громкость приема ее передач постепенно уменьшается, и, наконец, прием совсем прекращается. Средние волны хуже огибают Землю и, кроме того, сильнее, чем длинные, поглощаются ею. Этим — то и объясняется меньшая «дальнобойность» СВ радиовещательных станций по сравнению с ДВ станциями. Так, например, сигналы радиостанции, работающей на волне длиной 300 — 400 м, могут быть приняты на расстоянии, в два — три раза меньшем, чем сигналы станции такой же мощности, но работающей на волне длиной 1500 — 2000 м. Чтобы повысить дальность действия СВ станций, приходится увеличивать их мощность. В вечернее и ночное время суток передачи ДВ и СВ радиостанций можно слышать на больших расстояниях, чем днем. Дело в том, что излучаемая вверх часть энергии радиоволн этих станций днем бесследно теряется в атмосфере. После же захода Солнца нижний слой ионосферы искривляет их путь так, что они возвращаются к Земле на таких расстояниях, на которых прием этих станций поверхностными волнами уже невозможен. Радиоволны коротковолнового диапазона сильно поглощаются Землей и плохо огибают ее поверхность. Поэтому уже на расстоянии в несколько десятков километров от таких радиостанций их поверхностные волны затухают. Но зато пространственные волны могут быть обнаружены приемниками на расстоянии в несколько тысяч километров от них и даже в противоположной точке Земли. Искривление пути пространственных коротких волн происходит в ионосфере. Войдя в ионосферу, они могут пройти в ней очень длинньй путь и вернуться на Землю далеко от радиостанции. Они могут совершить кругосветное путешествие — их можно принять даже в том месте, где расположена передающая станция. Этим и объясняется секрет хорошего распространения коротких волн на большие расстояния даже при малых мощностях передатчика. Но при распространении коротких волн могут образовываться зоны, где передачи KB радиостанции вообще не слышны. Их называют зонами молчания (см. рис. 4). Протяженность зоны молчания зависит от длины волны и состояния ионосферы, которое в свою очередь зависит от интенсивности солнечного излучения. Ультракороткие волны по своим свойствам наиболее близки к световым лучам. Они в основном распространяются прямолинейно и сильно поглощаются землей, растительным миром, различными сооружениями, предметами. Поэтому уверенный прием сигналов УКВ станций поверхностной волной возможен главным образом лишь тогда, когда между антеннами передатчика и приемника можно мысленно провести прямую линию, не встречающую по всей длине каких — либо препятствий в виде гор, возвышенностей, лесов. Ионосфера для УКВ подобно стеклу для света — «прозрачна». Ультракороткие волны почти беспрепятственно проходят через нее. Поэтому этот диапазон радиоволн используют для связи с искусственными спутниками Земли и космическими кораблями.

Как работает радиоприемник?

В любом простейшем радиовещательном приемнике, независимо от его сложности, совершенно обязательно есть три элемента, обеспечивающие ему работоспособность. Эти элементы — колебательный контур, детектор и телефоны или, если приемник с усилителем 3Ч, динамическая головка прямого излучения. Колебательный контур: — устройство простейшего колебательного контура и его схема изображены на рис. 5. Он, как видите, состоит из катушки L и конденсатора С, образующих замкнутую электрическую цепь в которой при определенных условиях в контуре могут возникать и существовать электрические колебания. Поэтому его и называют колебательным контуром.

Рис. 5 Простейший электрический колебательный контур.

Приходилось ли вам наблюдать такое явление: в момент выключения питания электроосветительной лампы между размыкающимися контактами выключателя появляется искра. Если случайно соединить выводы полюсов батареи электрического карманного фонарика (чего нужно избегать), в момент их разъединения между ними также проскакивает маленькая искра. А на заводах, в цехах фабрик, где рубильниками разрывают электрические цепи, по которым текут токи большой силы, искры могут быть столь значительными, что приходится принимать меры, чтобы они не причинили вреда человеку, включающему ток. Почему возникают эти искры? Вы уже знаеште, что вокруг проводника с током существует магнитное поле, которое можно изобразить в виде замкнутых магнитных силовых линий, пронизывающих окружающее его пространство. Обнаружить это, поле, если оно постоянное, можно с помощью магнитной стрелки компаса. Если отключить проводник от источника тока, то его исчезающее магнитное поле, рассеиваясь в пространстве, будет индуцировать токи в ближайших от него других проводниках. Ток индуцируется и в том проводнике, который создал ЭДС, магнитное поле. А так как он находится в самой гуще своих же магнитных силовых линий, в нем будет индуцироваться более сильный ток, чем в любом другом проводнике. Направление этого тока будет таким же, каким оно было в момент разрыва проводника. Иначе говоря, исчезающее магнитное поле будет поддерживать создающий его ток до тех пор, пока оно само не исчезнет, т.е. полностью не израсходуется содержащаяся в нем энергия. Следовательно, ток в проводнике течет и после того, как выключен источник тока, но, разумеется, недолго — ничтожно малую долю секунды. После размыкания цепи, электрический ток может некоторое время течь через воздушный промежуток между разъединенными концами проводника, между контактами выключателя или рубильника. Вот этот ток через воздух и образует электрическую искру. Это явление называют самоиндукцией, а электрическую силу (не путайте с явлением индукции, знакомым вам по предыдущим урокам), которая под действием ее исчезающего магнитного поля поддерживает в нем ток, — электродвижущей силой самоиндукции или, сокращенно, ЭДС самоиндукции. Чем больше ЭДС самоиндукции, тем значительнее может быть искра в месте разрыва электрической цепи. Явление самоиндукции наблюдается не только при выключении, но и при включении тока. В пространстве, окружающем проводник, магнитное поле возникает сразу при включении тока, вначале оно слабое, но затем очень быстро усиливается. Усиливающееся магнитное поле тока также возбуждает ток самоиндукции, но этот ток направлен навстречу основному току. Ток самоиндукции мешает мгновенному увеличению основного тока и росту магнитного поля. Однако через короткий промежуток времени основной ток в проводнике преодолевает встречный ток самоиндукции и достигает наибольшего значения, магнитное поле становится постоянным и действие самоиндукции прекращается. Явление самоиндукции можно сравнивать с явлением инерции. Санки, например, трудно сдвинуть с места. Но когда они наберут скорость, запасутся кинетической энергией — энергией движения, их невозможно остановить мгновенно. При торможении санки продолжают скользить до тех пор, пока запасенная ими энергия движения не израсходуется на преодоление трения о снег. Все ли проводники обладают одинаковой самоиндукцией? Нет Чем длиннее проводник, тем значительнее самоиндукция. В проводнике, свернутом в катушку, явление самоиндукции сказывается сильнее, чем в прямолинейном проводнике, так как магнитное поле каждого витка катушки наводит ток не только в этом витке, но и в соседних витках этой катушки. Чем больше длина провода в катушке, тем дольше будет существовать в нем ток самоиндукции после выключения основного тока. И наоборот, потребуется больше времени после включения основного тока, чтобы ток в цепи увеличился до определенного значения и установилось постоянное по силе магнитное поле. Запомните: свойство проводника влиять на ток в цепи и изменении его значения называют индуктивностью, а катушки, в которых наиболее сильно проявляется это свойство — катушками самоиндукции или индуктивности. Чем больше число витков и размеры катушки, тем больше ее индуктивность, тем значительнее влияет она на ток в электрической цепи. Итак, катушка индуктивности препятствует как нарастанию, так и убыванию тока в электрической цепи. Еше она находится в цепи постоянного тока, и влияние ее сказывается только при включении и выключении тока. В цепи же переменного тока, где беспрерывно изменяются ток и его магнитное поле ЭДС самоиндукции катушки действует все время, пока течет ток. Это электрическое явление и используется в первом элементе колебательного контура приемника — катушке индуктивности. Вторым элементом колебательного контура приемника является накопитель электрических зарядов — конденсатор. Простейший конденсатор представляет собой два проводника электрического тока, например: — две металлические пластины, называемые обкладками конденсатора, разделенные диэлектриком например: — воздухом или бумагой. Чем больше площадь обкладок конденсатора и чем ближе они расположены друг к другу, тем больше электрическая емкость этого прибора. Если к обкладкам конденсатора подключить источник постоянного тока (рис. 6, а), то в образовавшейся цепи возникнет кратковременный ток и конденсатор зарядится до напряжения, равного напряжению источника тока. Вы можете спросить: почему в цепи где есть диэлектрик, возникает ток? Когда мы присоединяем к конденсатору источник тока, электроны в проводниках образовавшейся цепи начинают двигаться в сторону положительного полюса источника тока, образуя кратковременный поток электронов во всей цепи. В результате обкладка конденсатора, которая соединена с положительным полюсом источника тока, обедняется свободными электронами и заряжается положительно, а другая обкладка обогащается свободными электронами и, следовательно, заряжается отрицательно. Как только конденсатор зарядится, кратковременный ток в цепи, называемый током зарядки конденсатора, прекратится.

Рис. 6 Процесс зарядки — разрядки конденсатора.

Если источник тока отключить от конденсатора, то конденсатор окажется заряженным (рис. 6, б). Переходу избыточных электронов с одной обкладки на другую препятствует диэлектрик. Между обкладками конденсатора тока не будет, а накопленная им электрическая энергия будет сосредоточена в электрическом поле диэлектрика. Но стоит обкладки заряженного конденсатора соединить каким — либо проводником (рис. 6, в), «лишние» электроны отрицательно заряженной обкладки перейдут по этому проводнику на другую обкладку, где их недостает, и конденсатор разрядится. В этом случае в образовавшейся цепи также возникает кратковременный ток, называемый током разрядки конденсатора. Если емкость конденсатора большая, и он заряжен до значительного напряжения, момент его разрядки сопровождается появлением значительной искры и треска. Свойство конденсатора накапливать электрические заряды и разряжаться через подключенные к нему проводники используется в колебательном контуре радиоприемника. А теперь, вспомните обыкновенные качели. На них можно раскачиваться так, что «дух захватывает». Что для этого надо сделать? Сначала подтолкнуть, чтобы вывести качели из положения покоя, а затем прикладывать некоторую силу, но обязательно только в такт с их колебаниями. Без особого труда можно добиться сильных размахов качелей — получить большие амплитуды колебаний. Даже маленький мальчик может раскачать на качелях взрослого человека, если будет прикладывать свою силу умеючи. Раскачав качели посильнее, чтобы добиться больших амплитуд колебаний, перестанем подталкивать их. Что произойдет дальше? За счет запасенной энергии они некоторое время свободно качаются, амплитуда их колебаний постепенно убывает, как говорят, колебания затухают, и, наконец, качели остановятся. При свободных колебаниях качелей, так же как свободно подвешенного маятника, — запасенная потенциальная энергия переходит в кинетическую энергию движения, которая в крайней верхней точке вновь переходит в потенциальную, а через долю секунды — опять в кинетическую. И так до тех пор, пока не израсходуется весь запас энергии на преодоление трения веревок в местах подвеса качелей и сопротивления воздуха. При сколь угодно большом запасе энергии свободные колебания всегда являются затухающими: с каждым колебанием их амплитуда уменьшается и колебания постепенно совсем затухают — качели останавливаются. Но период, т. е. время, в течение которого происходит одно колебание, а значит, и частота колебаний остаются постоянными. Однако, если качели все время подталкивать в такт с их колебаниями и тем самым пополнять потери энергии, расходуемой на преодоление различных тормозящих сил, колебания станут незатухающими. Это уже не свободные, а вынужденные колебания. Они будут длиться до тех пор, пока не перестанет действовать внешняя подталкивающая сила. Я вспомнил здесь о качелях потому, что физические явления, происходящие в такой механической колебательной системе, очень схожи с явлениями в электрическом колебательном контуре. Чтобы в контуре возникли электрические колебания, ему надо сообщить энергию, которая «подтолкнула» бы в нем электроны. Это можно сделать, зарядив, например, его конденсатор. Разорвем выключателем S колебательный контур и подключим к обкладкам его конденсатора источник постоянного тока, как показано на (рис. 7 слева). Конденсатор зарядится до напряжения батареи GB. Затем отключим батарею от конденсатора контур замкнем выключателем S.

Рис. 7 Электрические колебания в контуре.

Явления, которые теперь будут происходить в контуре, изображены графически на (рис. 7 справа). В момент замыкания контура выключателем верхняя обкладка конденсатора имеет положительный заряд, а нижняя — отрицательный (рис. 7, а). В это время (точка 0 на графике) тока в контуре нет, а вся энергия, накопленная конденсатором, сосредоточена в электрическом поле его диэлектрика. При замыкании конденсатора на катушку конденсатор начнет разряжаться. В катушке появляется ток, а вокруг ее витков — магнитное поле. К моменту полной разрядки конденсатора (рис. 7, б), отмеченному на графике цифрой 1, когда напряжение на его обкладках уменьшится до нуля, ток в катушке и энергия магнитного поля достигнут наибольших значений. Казалось бы, что в этот момент ток в контуре должен был прекратиться. Этого, однако, не произойдет, так как от действия ЭДС самоиндукции, стремящейся поддержать ток, движение электронов в контуре будет продолжаться. Но только до тех пор, пока не израсходуется вся энергия магнитного поля. В катушке в это время будет течь убывающий по значению, но первоначального направления индуцированный ток. К моменту времени, отмеченному на графике цифрой 2, когда энергия магнитного поля израсходуется, конденсатор вновь окажется заряженным, только теперь на его нижней обкладке будет положительный заряд, а на верхней — отрицательный (рис. 7, в). Теперь электроны начнут обратное движение — в направлении от верхней обкладки через катушку к нижней обкладке конденсатора. К моменту 3 (рис. 7, г) конденсатор разрядится, а магнитное поле катушек достигнет наибольшего значения, И опять ЭДС самоиндукции «погонит» по проводу катушки электроны, перезаряжая тем самым конденсатор. В момент времени 4 (рис. 7, д) состояние электронов в контуре будет таким же, как в первоначальный момент — 0. Закончилось одно полное колебание. Естественно, что заряженный конденсатор вновь будет разряжаться на катушку, перезаряжаться и произойдут второе, за ним третье, четвертое колебания. Другими словами, в контуре возникнет переменный электрический ток, электрические колебания. Но этот колебательный процесс в контуре не бесконечен. Он продолжается до тех пор пока вся энергия, полученная конденсатором от батареи, не израсходуется на преодоление сопротивления провода катушки контура. Колебания контура свободные и, следовательно, затухающие. Какова частота таких колебаний электронов в контуре? Чтобы подробнее разобраться в этом вопросе, советую провести такой опыт с простейшим маятником. Подвесьте на нитке длиной 100 см. шарик, слепленный из пластилина, или иной груз массой в 20 — 40 г. (на рис. 8 длина маятника обозначена латинской буквой L).

Рис. 8 Графики колебаний простейшего маятника.

Выведите маятник из положения равновесия и пользуясь часами с секундной стрелкой, сосчитайте, сколько полных колебаний он делает за 1 мин. Примерно 30. Следовательно, частота колебаний этого маятника равна 0,5 Гц, а период 2 с. За период потенциальная энергия маятника дважды переходит в кинетическую, а кинетическая в потенциальную. Укоротите нить наполовину. Частота маятника увеличится примерно в полтора раза и во столько же раз уменьшится период колебаний. Этот опыт позволяет сделать вывод: с уменьшением длины маятника частота его собственных колебаний увеличивается, а период пропорционально уменьшается. Изменяя длину подвески маятника, добейтесь, чтобы его частота колебаний равнялась 1 Гц. Это должно быть при длине нити около 25 см. При этом период колебаний маятника будет равен 1 с. Каким бы вы не пытались создать первоначальный размах маятника, частота его колебаний будет неизменной. Но стоит только укоротить или удлинить нитку, как частота колебаний сразу изменится. При одной и той же длине нитки всегда будет одна и та же частота колебаний. Это собственная частота колебаний маятника. Получить заданную частоту колебаний можно, подбирая длину нити. Колебания нитяного маятника — затухающие. Они могут стать незатухающими только в том случае, если маятник в такт с его колебаниями слегка подталкивать, компенсируя таким образом ту энергию, которую он затрачивает на преодоление сопротивления, оказываемого ему воздухом, энергию трения, земного притяжения. Собственная частота характерна и для электрического колебательного контура. Она зависит, во — первых, от индуктивности катушки. Чем больше число витков и диаметр катушки, тем больше ее индуктивность, тем больше будет длительность периода каждого колебания. Собственная частота колебаний в контуре будет соответственно меньше. И, наоборот, с уменьшением индуктивности катушки сократится период колебаний — возрастет собственная частота колебаний в контуре. Во — вторых, собственная частота колебаний в контуре зависит от емкости его конденсатора. Чем емкость больше, тем больший заряд может накопить конденсатор, тем больше потребуется времени для его перезарядки, тем меньше частота колебаний в контуре. С уменьшением емкости конденсатора частота колебаний в контуре возрастает. Таким образом, собственную частоту затухающих колебаний в контуре можно регулировать изменением индуктивности катушки или емкости конденсатора. Но в электрическом контуре, как и в механической колебательной системе, можно получить и незатухающие, т.е. вынужденные колебания, если при каждом колебании пополнять контур дополнительными порциями электрической энергии от какого — либо источника переменного тока. Каким же образом в контуре приемника возбуждаются и поддерживаются незатухающие электрические колебания? Колебания радиочастоты, возбуждающиеся в антенне приемника. Эти колебания сообщают контуру первоначальный заряд, они же и поддерживают ритмичные колебания электронов в контуре. Но наиболее сильные незатухающие колебания в контуре приемника возникают только в момент резонанса собственной частоты контура с частотой тока в антенне. Как это понимать? Люди старшего поколения рассказывают, будто в Петербурге от шедших в ногу солдат обвалился Египетский мост. А могло это случиться, видимо, при таких обстоятельствах. Все солдаты ритмично шагали по мосту. Мост от этого стал раскачиваться — колебаться. По случайному стечению обстоятельств собственная частота колебаний моста совпала с частотой шага солдат, и мост, как говорят, вошел в резонанс. Ритм строя сообщал мосту все новые и новые порции энергии. В результате мост настолько раскачался, что обрушился: слаженность воинского строя нанесла вред мосту. Если бы резонанса собственной частоты колебаний моста с частотой шага солдат не было, с мостом ничего бы не случилось. Поэтому, между прочим, при прохождении солдат по слабым мостам принято подавать команду «сбить ногу». А вот опыт. Подойдите к какому — нибудь струнному музыкальному инструменту и громко крикните «а»: какая — то из струн отзовется — зазвучит. Та из них, которая окажется в резонансе с частотой этого звука, будет колебаться сильнее остальных струн — она — то и отзовется на звук. Еще один опыт с маятником. Натяните горизонтально нетолстую веревку. Привяжите к ней тот же маятник из нити и пластилина (рис.9).

Рис. 9 Опыт, иллюстрирующий явление резонанса.

Перекиньте через веревку еще один такой же маятник, но с более длинной ниткой. Длину подвески этого маятника можно изменять, подтягивая рукой свободный конец нитки. Приведите маятник в колебательное движение. При этом первый маятник тоже станет колебаться, но с меньшей амплитудой. Не останавливая колебаний второго маятника, постепенно уменьшайте длину его подвески — амплитуда колебаний первого маятника будет увеличиваться. В этом опыте, иллюстрирующем резонанс механических колебаний, первый маятник является приемником колебаний, возбуждаемых вторым маятником. Причиной, вынуждающей первый маятник колебаться, являются периодические колебания растяжки с частотой, равной частоте колебаний второго маятника. Вынужденные колебания первого маятника будут иметь максимальную амплитуду, а его собственная частота совпадает с частотой колебаний второго. Такие или подобные явления, только, разумеется, электрического происхождения, наблюдаются и в колебательном контуре приемника. От действия волн многих радиостанций в приемной антенне возбуждаются токи самых различных частот. Нам из всех колебаний радиочастот надо выбрать только несущую частоту той радиостанции, передачи которой мы хотим слушать. Для этого следует так подобрать число витков катушки и емкость конденсатора колебательного контура, чтобы его собственная частота совпадала с частотой тока, создаваемого в антенне радиоволнами интересующей нас станции. В этом случае в контуре возникнут наиболее сильные колебания с несущей частотой той радиостанции, на волну которой он настроен. Это и есть настройка контура приемника в резонанс с частотой передающей станции. При этом сигналы других станций совсем не слышны или прослушиваются очень тихо, так как возбуждаемые ими колебания в контуре будут во много раз более слабыми. Таким образом, настраивая контур своего приемника в резонанс с несущей частотой радиостанции, вы с его помощью как бы отбираете, выделяя колебания частоты , только этой станции. Чем лучше контур будет выделять нужные колебания из антенны, тем выше селективность приемника, тем слабее будут помехи со стороны других радиостанций. До сих пор я рассказывал вам о замкнутом колебательном контуре, т.е. контуре, собственная частота которого определяется только индуктивностью катушки и емкостью конденсатора образующих его. Однако во входной контур приемника входят также антенна и заземление. Это уже не замкнутый, а открытый колебательный контур. Дело в том, что провод антенны и земля являются «обкладками» конденсатора, обладающего некоторой электрической емкостью. В зависимости от длины провода и высоты антенны над землей эта емкость может составлять несколько сотен пикофарад. Но ведь антенну и землю можно рассматривать и как не полный виток большой катушки. Стало быть, антенна и заземление, взятые вместе, обладают еще и индуктивностью. А емкость совместно с индуктивностью образуют колебательный контур (рис. 10).

Рис. 10 Антена и зазамление — открытый колебательный контур.

Такой контур, являющийся открытым колебательным контуром, тоже обладает собственной частотой колебаний. Включая между антенной и землей катушки индуктивности и конденсаторы, мы можем изменять его собственную частоту, настраивать его в резонанс с частотами разных радиостанций. Как это делается на практике, вы уже знаете. Я не ошибусь, если скажу, что колебательный контур является «сердцем» радиоприемника. И не только радиоприемника. Поэтому ему я и уделил побольше внимания. Перехожу ко второму элементу приемника — детектору.

Детектор и детектирование радиосигнала

Детектор — двухэлектродный полу — проводниковый прибор (высокочастотный диод), обладающий односторонней электропроводностью: хорошо проводит ток одного направления и не проводит или слабо проводит — ток обратного направления. Для простоты объяснения работы диода как детектора будем считать, что ток обратного направления он вообще не проводит и является для него как бы изолятором. Это свойство диода иллюстрирует график, изображенный на (рис. 11), диод беспрепятственно пропускает через себя положительные полуволны переменного тока и совсем не пропускает отрицательные полуволны. Отрицательные полуволны диод как бы срезает. В результате такого действия диода переменный ток преобразуется в пульсирующий ток одного направления, но изменяющийся по величине с частотой пропускаемого через него тока. Этот преобразовательный процесс, называемый выпрямлением переменного тока, лежит в основе детектирования принятых радиосигналов.

Рис. 11 Диод преобразующий переменный ток в пульсирующий.

Посмотрите на графики, показанные на (рис. 12). Они иллюстрируют процессы, происходящие в детекторной цепи простейшего приемника. Под действием радиоволн в контуре приемника возбуждаются модулированные колебания радиочастоты (рис. 12, а). К контуру подключена цепь, состоящая из диода и телефонов.

Рис. 12 Графики иллюстрирующие детектирование модулированных колебаний радиочастоты.

Для этой цепи колебательный контур является источником переменного тока радиочастоты. Поскольку диод пропускает ток только одного направления, то модулированные колебания радиочастоты, поступающие в его цепь, будут им выпрямлены (рис. 12,б), говоря иначе, продетектированы. Если провести штриховую линию, огибающую верщины выпрямленного тока, то получится «рисунок» тока звуковой частоты, которым модулирован ток, поступающий в антенну радиостанции во время передачи. Ток, получившийся в результате детектирования состоит из импульсов радиочастоты, амплитуды которых изменяются со звуковой частотой. Его можно рассматривать как суммарный ток и разложить на две составляющие: высокочастотную и низкочастотную. Их называют соответственно высокочастотной и составляющей звуковой частоты пульсирующего тока. В простейщем приемнике составляющая звуковой частоты идет через телефоны и преобразуется ими в звук.

Головной телефон и его устройство

Телефон — третье, последнее звено простейшего приемника, которое, образно выражаясь, «выдает готовую продукцию» — звук. Это один из старейших электротехнических приборов, почти без изменения сохранивший свои основные черты до наших дней. Для детекторных и многих простейших транзисторных приемников используют головные телефоны, например типов ТОН-1, ТГ-1, ТА-4. Это два последовательно соединенных телефона, удерживающихся на оголовье. Отвернем крышку одного из телефонов (рис. 13, а).

Рис. 13 Устройство электромагнитного телефона.

Под нею находится круглая жестяная пластинка — мембрана. Сняв осторожно мембрану, мы увидим две катушки, насаженные на полюсные наконечники постоянного магнита, впрессованного в корпус. Катушки соединены последовательно, а крайние выводы припаяны к стерженькам, к которым с наружной стороны при помощи прижимных винтов подключен шнур с однополюсными штепсельными вилками. Как работает телефон? Мембрана, издающая звук, находится возле полюсных наконечников магнита и операется на бортики корпуса (рис. 13, а). Под действием поля магнита она немного прогибается в середине, но неприкасается к полюсным наконечникам магнита (на рис. 13, б) — сплошная линия. Когда через, катушки телефона течет ток, он создает вокруг катушек магнитное поле, которое взаимодействует с магнитным полем постоянного магнита. Сила этого магнитного поля, а значит и сила притяжения мембраны к полюсным наконечникам зависит от направления тока в катушках. При одном направлении, когда направления магнитных силовых линий катушек и магнита совпадают и их поля складываются, мембрана сильнее притягивается к полюсам магнита (на рис. 13, б — нижняя штриховая линия). При другом направлении тока силовые линии катушки и магнита направлены встречно и общее поле становится слабее, чем поле магнита. В этом случае мембрана слабее притягивается полюсными наконечниками и выпрямляясь, несколько удаляется от них (рис. 13, б — верхняя штриховая линия). Если через катушки телефона пропускать переменный ток звуковой частоты, суммарное магнитное поле станет то усиливаться, то ослабляться а мембрана будет то приближаться к полюсным наконечникам магнита, то отходить от них, т. е. колебаться с частотой тока. Колеблясь, мембрана создаст в окружающем пространстве звуковые волны. С первого взгляда может показаться что постоянный магнит в телефоне не нужен: катушки можно надеть на железную ненамагниченную подковку. Но это не так. И вот почему. Железная подковка, намагничиваемая переменным током будет притягивать мембраму независимо от того, идет ли ток через катушки в одном направлении или другом. Значит, за один период переменного тока мембрана притянется во время первого полупериода, отойдет от него и еще раз притянется во время второго полупериода, т.е. на один период переменного тока (рис, 14, а) она сделает два колебания (рис. 14, б).

Если, например, частота тока 500 Гц, то мембрана телефона за 1 с сделает 500 * 2 = 1000 колебаний и тон звука исказится — будет вдвое выше. Вряд ли нас устроит такой телефон. С постоянным же магнитом дело обстоит иначе: при одном полупериоде происходит усиление магнитного поля — уже притянутая мембрана прогнется еще больше; при другом полупериоде поле ослабевает и мембрана, выпрямляясь, отходит дальше от полюсов магнита. Таким образом, при наличии постоянного магнита мембрана за один период переменного тока делает только одно колебание (рис. 14, в) и телефон не искажает звук. Постоянный магнит, кроме того, повышает громкость звучания телефона. Теперь разберем такой вопрос: зачем параллельно головным телефонам подключают блокировочный конденсатор? Какова его роль? Электрическая емкость блокировочного конденсатора такова, что через него свободно проходят токи высокой частоты, а токам звуковой частоты он оказывает значительное сопротивление. Телефоны, наоборот, хорошо пропускают токи звуковой частоты и оказывают большое сопротивление токам высокой частоты. На этом участке детекторной цепи высокочастотный пульсирующий ток разделяется (на рис. 15 — в точке а) на составляющие, которые далее идут: высокочастотная — через блокировочный конденсатор Сбл, а составляющая звуковой частоты через телефоны. Затем составляющие соединяются (на рис. 15 — в точке б) и далее опять идут вместе.

Рис. 15 В точке а детекторной цепи составляющие пульсирующего тока разделяются, а в точке б, соединяются.

Назначение блокировочного конденсатора можно объяснить еще так. Телефон из — за инертности мембраны не может отзываться на каждый высокочастотный импульс тока в детекторной цепи. Значит, чтобы телефон работал, надо как — то «сгладить» высокочастотные импульсы, «заполнить» провалы тока между ними. Эта задача и решается с помощью блокировочного конденсатора следующим образом. Отдельные высокочастотные импульсы заряжают конденсатор. В моменты между импульсами конденсатор разряжается через телефон, заполняя таким образом «провалы» между импульсами. В результате через телефон идет ток одного направления, но изменяющийся по величине со звуковой частотой, который и преобразуется им в звук. Еще короче о роли блокировочного конденсатора можно сказать так: он фильтрует сигнал звуковой частоты, выделенный диодом, т. е. «очищает» его от составляющей радиочастоты. Качество работы телефона оценивают главным образом с точки зрения его чувствительности — способности реагировать на слабые колебания электрического тока. Чем слабее колебания, на которые отзывается телефон, тем выше его чувствительность. Чувствительность телефона зависит от числа витков в его катушках и качества магнита. Два телефона с совершенно одинаковыми магнитами, но с катушками, содержащими неодинаковое число витков, различны по чувствительности. Лучшей чувствительностью будет обладать тот из них, в котором использованы катушки с большим числом витков. Чувствительность телефона зависит также от положения мембраны относительно полюсных наконечников магнита. Наилучшая чувствительность его будет в том случае, когда мембрана находится очень близко к полюсным наконечникам, но, вибрируя, не прикасается к ним. Телефоны принято подразделять на высокоомные — с большим числом витков в катушках, и низкоомные — с относительно небольшим числом витков. Для детекторного приемника пригодны только высокоомные телефоны. Катушки каждого телефона типа ТОН-1, например, намотаны эмалированным проводом толщиной 0,06 мм и имеют по 4000 витков. Их сопротивление постоянному току около 2200 Ом. Это число, характеризующее телефоны, выштамповано на их корпусах. Поскольку два телефона соединены последовательно, их общее сопротивление постоянному току составляет 4400 Ом. Сопротивление постоянному току низкоомных телефонов, например типа ТА — 56, может быть 50 — 60 Ом. Низкоомные телефоны можно использовать для некоторых транзисторных приемников. Как проверить исправность и чувствительность головных телефонов? Прижмите их к ушам. Смочте слюной штепсельные вилки на конце шнура, а затем коснитесь ими друг друга — в телефонах должен быть слышен слабый щелчок. Чем сильнее этот щелчок, тем чувствительнее телефоны. Щелчки получаются потому, что смоченный контакт между металлическими вилками представляет собой очень слабый источник тока. Грубо проверить телефоны можно с помощью батареи для карманного электрического фонарика. При подключении телефонов к батарее и отключении от нее должны быть слышны резкие щелчки. Если щелчков нет, значит, где — то в катушках или шнуре имеется обрыв или плохой контакт.

Практическая работа

В этой практической работе мы сконструируем простейший радиоприемник (детекторный приемник), без которого, на мой взгляд, немыслимо дальнейшее освоение любой радиоприемной аппаратуры. Спросите у любого специалиста в области радиоэлектроники (КВ — УКВ радиосвязи), что такое детекторный радиоприемник и я думаю, он без промедления даст вам вразумительный ответ. Одним словом это классика, основа — основ, с чего начинали наши отцы и деды. И мы от них постараемся не отставать.

Главное достоинство такого варианта простейшего радиоприемника заключается в том, что в нем легко делать любые изменения и дополнения, исправлять ошибки путем переключения соединительных проводников, поскольку все его детали будут лежать перед вами в развернутом виде. Опыты с ним помогут вам понять основные принципы работы любого радиовещательного приемника и получить некоторые практические навыки радиотехнического конструирования. Для такого приемника понадобятся: катушка индуктивности, стержень из феррита марки 400НН или 600НН диаметром 7 — 8 мм и длиной 120 — 140 мм (такие стержни используют для магнитных антенн транзисторных приемников), полупроводниковый точечный диод, который в приемнике будет детектором, несколько конденсаторов постоянной емкости и головные телефоны (рис. 1).

Рис. 1 Самодельная катушка индуктивности (а), феритовый стержень (б), точечный диод (в), конденсаторы (г) и головные телефоны (д), необходимые для опытного приемника.

Катушку индуктивности сделайте сами (по предыдущим урокам вы знаете как это делается). Остальные детали готовые. Диод может быть любым из серий Д9, Д2. Конденсаторы также любых типов — слюдяные, керамические или бумажные емкостью от нескольких десятков до нескольких тысяч пикофарад (сокращенно: пФ). Головные телефоны высокоомные, т. е. с обмотками сопротивлением 1500 — 2200 Ом, например типа ТОН — 1 или ТА — 4. Несколько позже, когда приступите к экспериментам, нужны будут некоторые другие детали и материалы. Для катушки потребуется обмоточный провод марки ПЭВ — 1 (Провод с Эмалевой Высокопрочной изоляцией в один слой), ПЭВ — 2 (то же, но с изоляцией в два слоя) или ПЭЛ (Провод с Эмалевой Лакостойкой изоляцией) диаметром 0,15 — 0,2 мм. Обмоточные провода этих марок и их диаметр обозначают так: ПЭВ — 1 0,15, ПЭВ — 2 0,18, ПЭЛ 0,2. Годятся обмоточные провода и других марок, например ПБД — с изоляцией из двух (буква Д) слоев хлопчатобумажной пряжи (буква Б), или ПЭЛШО — с эмалевой лакостойкой изоляцией и одним (буква О) слоем натурального шелка (буква Ш). Важно лишь, чтобы изоляция провода была непопорченной, иначе между витками катушки может возникнуть замыкание, чего допускать нельзя. Внутренний диаметр каркаса катушки, склеенный из писчей бумаги в 3 — 4 слоя, должен быть таким, чтобы в него с небольшим трением входил ферритовый стержень. Прежде чем наматывать катушку, вставьте в каркас стержень. Провод сильно не натягивайте, иначе каркас сожмется и из него будет трудно вытащить стержень. Всего на каркас надо намотать в один ряд 300 витков провода, делая через каждые 50 витков отводы в виде петель. Получится однослойная шести — секционная катушка индуктивности с двумя крайними выводами и пятью отводами. Чтобы крайние витки провода готовой катушки не спадали, закрепите их на каркасе колечками, нарезанными из резиновой или поливинилхлоридной трубки, или обмотайте нитками. Дополнительно витки провода катушки можно скрепить тонким слоем клея «Момент». Концы каркаса аккуратно подрежьте острым ножом. Бывает, что во время намотки катушки провод оборвется или одного отрезка провода не хватит на всю катушку. В таком случае концы провода, которые нужно соединить, должны быть очищены от изоляции, крепко скручены, пропаяны и обязательно обмотаны тонкой изоляционной лентой. Если соединение приходится возле отвода, то лучше не жалеть нескольких витков провода и сделать его в петле. Вот теперь, приступайте к сборке своего первого радиоприемника (рис. 2).

Рис. 2 Соединение деталей опытного приемника.

Концы выводов и отводов, катушки зачистить от изоляции, только осторожно, чтобы не порвать провод. Один из крайних выводов назовем началом катушки и обозначим буквой (н). Соедините его с диодом. Второй крайний вывод катушки, ее конец (к), соедини с одним из контактных штырьков шнура головных телефонов. Оставшиеся свободными вывод диода и штырек телефонов тоже соедините между собой. К проводнику, идущему от начала катушки к диоду, прочно прикрутите провод антенны, предварительно зачистив его от изоляции. Этот проводник приемника будем называть антенным. К проводнику, соединяющему конец катушки с телефонами, прикрутите провод заземления. Это будет заземленный проводник. Во время опытов его придется переключать с одного вывода катушки на другой (на рис. 2 показано штриховой линией со стрелкой), не изменяя при этом соединения заземления с телефонами. Совершим «прогулку» по цепям получившегося приемника. От начала катушки (н) по антенному проводнику мы попадаем к диоду, а от него — к головным телефонам. Через телефоны, далее по заземленному проводнику и через все витки катушки приходим к отправной точке (н). Получилась замкнутая электрическая цепь, состоящая из катушки, диода и телефонов. Ее называют детекторной. Если в этой цепи где — либо окажется обрыв, плохой контакт между деталями или соединительными проводниками, например непрочная скрутка, приемник, естественно, работать не будет. Кратчайший путь из антенны в землю — через катушку. По этому пути пойдет ток высокой частоты, возбуждаемый в антенне радиоволнами. Этот ток создаст на концах катушки высокочастотное напряжение, которое вызовет ток такой же частоты во всей детекторной цепи. Цепь, состоящую из антенны, катушки и заземления называют антенной или антенным контуром. Обратите внимание: контурная катушка приемника входит как в антенную, так и в детекторную цепи. После такой прогулки по цепям приемника можно перейти к его испытанию. Наденьте на голову телефоны, прижмите их плотнее к ушам, прислушайтесь. Возможно, что сразу вы ничего не услышите даже при заведомо хороших антенне и заземлении, предварительно проверенных диоде и телефонах. Это потому, что приемник, видимо, не настроен на несущую частоту радиовещательной станции, сигналы которой хорошо слышны в вашем районе, или вы попали в перерыв передачи. Настраивать такой приемник можно изменением числа витков катушки, включаемых в антенный контур. На (рис. 2) в антенный контур включены все 300 витков катушки. Если заземленный проводник отсоединить от конца катушки и присоединить, например, к отводу 5, то в контур будет включено уже не 300, а 250витков. Если же этот проводник переключить на отвод 4, в контур будет включено 200 витков. При переключении его на отвод 3 в антенный контур будет включено 150 витков и т.д. При этом нижние секции окажутся не включенными в контур и в работе приемника участвовать не будут. Таким образом, переключением заземленного проводника вы можете включать в контур разное число витков через 50 витков. Запомните: чем больше длина волны радиовещательной станции, на которую можно настроить приемник, тем большее число витков катушки должно быть включено в антенный контур. Ваш опытный приемник можно настраивать на радиовещательные станции как средневолнового, так и длинноволнового диапазонов. Но, разумеется, передачи не всякой станции вы можете принять. На слабые сигналы отдаленных станций детекторный приемник реагировать не сможет — мала чувствительность. Теперь займитесь настройкой приемника путем присоединения заземленного проводника сначала к отводу 5, затем к отводу 4 и так до отвода 1. Одновременно следите, чтобы отводы катушки и соединительные проводники не соприкасались, а контакты в скрутках не нарушались. Иначе приемник совсем не будет работать или в телефонах будут слышны трески, шорохи, мешающие приему. Электрические контакты будут надежнее, если места соединений проводников и деталей пропаять. Настроив приемник на одну станцию, запомните число витков, включенных в контур, при котором станция слышна с наибольшей громкостью. Потом попытайтесь «найти» таким же способом другую станцию. Надеюсь, что вы добились некоторого успеха. Попробуте улучшить работу приемника. Не изменяя настройки приемника, присоедините параллельно телефонам (между его контактными штырьками) конденсатор. Емкость этого конденсатора, называемого в данном случае блокировочным, может быть от 1000 до 3000 пФ. При этом громкость звучания телефонов должна несколько увеличиться. А если радиовещательные станции находятся более чем в 150 — 200 км от того места, где вы живете, блокировочный конденсатор включайте в самом начале опыта. Способ настройки приемника только скачкообразным изменением числа витков катушки очень прост. Но он не всегда позволяет настроить приемник точно на несущую частоту станции. Точной настройки можно добиться дополнительным способом, например, с помощью гвоздя. Попробуйте настроить приемник уже знакомым вам способом на волну радиостанции и введите внутрь каркаса катушки толстый гвоздь или подходящего диаметра железный стержень. Что получилось? Громкость приема немного возрастет или, наоборот уменьшится. Вытащите гвоздь из катушки — громкость станет прежней. Теперь медленно вводите гвоздь в катушку и так же медленно извлекате его из катушки — громкость работы приемника будет немного, но плавно изменяться. Опытным путем, можно найти такое положение металлического предмета в катушке, при котором громкость звучания будет наилучшей. Этот опыт позволяет сделать вывод, что металлический стержень, помещенный в катушку, влияет на настройку контура. С таким способом настройки приемника, только, разумеется, с применением лучшего, чем гвозь, ферромагнитного сердечника, вы познакомитесь в дальнейшем. А пока предлагаю следующий опыт — настроить приемник на сигналы радиовещательной станции с помощью конденсатора переменной емкости. Для удобства проведения этого и не скольких последующих опытов с детекторным приемником, на фанерной дощечке размерами примерно 30 х 70 мм смонтируйте колодку со штепсельными гнездами, два зажима, блокировочный конденсатор, соединив их под дощечкой, как показано на рис. 3.

Рис. 3 Настройка приемника самодельным конденсатором переменной емкости.

Колодку с гнездами устанавите на дощечке так: просверлите в ней два отверстия диаметром 6 — 8 мм с расстоянием 20 мм между центрами и вставьте в них «хвосты» штепсельных гнезд. Колодку укрепите на дощечке шурупами или винтами с гайками. Начало катушки и антенну подключите к зажиму, с которым соединен диод, а ко второму зажиму, соединенному с гнездом телефонов, подключите конец катушки и заземление. Конденсатор переменной емкости может быть как с воздушным, так и с твердым диэлектриком. Но функцию конденсатора переменной емкости могут выполнять две металлические пластины размерами примерно 150 х 150 мм, вырезанные, например, из жести больших консервных банок. К пластинам припаяйте проводники длиной по 250 — 300 мм. При помощи этих проводников одну пластину соедините с зажимом антенны, а другую — с зажимом заземления. Положите пластины на стол одну возле другой, но так, чтобы они не соприкасались, и настройте приемник на радиостанцию только переключением секций катушки заземленным проводником. Теперь поднесите заземленную пластину к пластине, соединенной с антенной. Если громкость будет увеличиваться, сближайте пластины и, наконец, положите одну пластину на другую, проложив между ними лист сухой бумаги (чтобы не было электрического контакта). Найдите такое взаимное расположение пластин, при котором будет точная настройка. Если же при сближении пластин громкость приема будет уменьшаться, переключите заземленный проводник на ближний к началу катушки отвод и вновь сближайте пластины, добиваясь наибольшей громкости. В этом опыте настройка приемника на несущую частоту радиостанции осуществлялась двумя способами: грубо — изменением индуктивности катушки путем переключения ее секций, точно — изменением емкости пластинчатого конденсатора. Запомните: индуктивность катушки и емкость конденсатора при настройке приемника на радиостанцию взаимосвязаны. Одну и ту же радиостанцию можно слушать при включении в антенный контур приемника большего числа витков, т. е. большей индуктивности катушки, но при меньшей емкости конденсатора, либо, наоборот, при меньшей индуктивности катушки, но большей емкости конденсатора. Теперь снова настройте приемник на какую — либо радиостанцию, запомните громкость приема передачи, а затем, не изменяя настройки, включите между антенной и антенным зажимом конденсатор емкостью 47 — 62 пФ (рис. 4).

Рис. 4 Конденсатор включенный в цепь антенны, улучшает селективность приемника.

Что получилось? Громкость приема несколько уменьшилась. Произошло это потому, что конденсатор, включенный в цепь антенны, изменил параметры всего контура. Подстройте контур конденсатором переменной емкости до прежней громкости звучания телефонов. Если до включения в контур дополнительного конденсатора во время приема одной станции прослушивалась еще какая — то другая, близкая по частоте радиостанция, теперь она будет слышна много слабее, а возможно, и совсем не будет мешать. Приемник стал четче выделять сигналы той станции, на которую настроен, или, как говорят, улучшилась его селективность, т. е. избирательность. Вместо конденсатора постоянной емкости включите между антенной и приемником конденсатор переменной емкости. С его помощью вы сможете не только изменять селективность приемника, но, возможно и настраивать его на разные станции. Следующий опыт — настройка приемника ферритовым стержнем (рис. 5).

Рис. 5 Приемник с настройкой ферритовым стержнем.

Пластинчатый конденсатор удалите, а вместо него между зажимами антенны и заземления, т. е. параллельно катушке, включите слюдяной или керамический конденсатор емкостью 120 — 150 пФ. Прижмите телефоны поплотнее к ушам, сосредоточьтесь и очень медленно вводите ферритовый стержень внутрь каркаса катушки. Постепенно углубляя стержень в катушку, вы должны услышать передачи всех тех радиовещательных станций, прием которых возможен в вашей местности на детекторный приемник. Чем длиннее волна радиостанции, тем глубже должен быть введен стержень в катушку. Опытным путем найдите такое положение стержня в катушке, при котором наиболее громко слышны сигналы станции, и сделайте на стержне соответствующую пометку карандашом. Пользуясь ею как делениями шкалы, вы сможете быстро настроить приемник на волну этой станции. Продолжая опыт с использованием ферритового стержня, подключите параллельно катушке другой конденсатор емкостью 390 — 470 пФ. Как это повлияло на настройку приемника? Громкость осталась прежней, но для настройки на ту же станцию стержень приходится меньше вводить в катушку. Совсем удалите конденсатор, оставив включенной только катушку. Что получилось? Чтобы настраивать приемник на ту же станцию, стержень надо глубже вводить в катушку. Какие выводы можно сделать, проведя эксперименты с таким вариантом детекторного приемника? Основных два. Во — первых, ферритовый стержень значительно сильнее, чем металлический предмет, влияет на индуктивность катушки, а значит и на настройку контура. Во — вторых, с помощью ферритового стержня можно плавно и точно настраивать контур приемника на желательную радиостанцию. Еще один эксперимент. Антенну и заземление отключите от приемника, между ними включите диод, а параллельно — телефоны без блокировочного конденсатора. Вот и весь приемник. Работает? Тихо, вероятно? К тому же, возможно, одновременно слышны передачи двух — трех радиовещательных станций. От такого приемника ожидать лучшего не следует. вы, наверное, заметили, что когда дотрагиваешься рукой до деталей или соединительных проводников, громкость работы немного изменяется. Это объясняется расстройкой антенного контура, вносимой в него электрической емкостью вашего тела.

Принципиальная электрическая схема детекторного приемника

Чтобы правильно соединить детали приемника, вы пользовались рисунками. На них катушку, телефоны, диод — детектор и другие детали, приборы и соединения вы видели такими, какими они выглядят в натуре. Это очень удобно для начала, пока приходится иметь дело с совсем простыми радиотехническими конструкциями, состоящими из малого числа деталей. Но если попытаться изобразить таким способом устройство современного приемника, то получилась бы такая «паутина» деталей и проводов, в которой невозможно было бы разобраться. Чтобы этого избежать, любой электроприбор или радиоустройтво изображают схематически, т. е. при помощи упрощенного чертежа — схемы. Различают три основных вида схем: структурные, принципиальные электрческие и схемы электрических соедининий. Структурная схема представляет собой упрощенный чертеж, на котором группы деталей и приборов, выполняющие определенные функции радиотехнического устройства, изображают условно прямоугольниками или иными символами. Структурная схема дает лишь общее представление о работе этот устройства, о его структуре и связях между его функциональными группами. Примером структурной схемы можем служить (рис. 2), по которому я рассказывал вам о принципе работы радиовещательной станции. Можно ли таким способом изобразить устройство детекторного приемника? Конечно, можно. Нарисуйте в один ряд четыре прямоугольника и соедините их между собой линиями со стрелками, идущими слева направо. В крайний левый прямоугольник впишите слово «Антенна», в следующий за ним прямоугольник — «Колебательный контур», в третий прямоугольник — «Детектор», в четвертый — «Телефоны». Получится структурная схема детекторного приемника. «Прочитать» ее можно так: модулированные колебания радиочастоты, возбужденные в антенне, поступают в колебательный контур приемника, а затем к детектору, детектор выделяет из принятого сигнала колебания звуковой частоты, которые телефоны преобразуют в звук. Раньше такие чертежи называли скелетными схемами или блок — схемами. Сейчас эта терминология считается устаревшей. Принципиальную электрическую схему чаще называют принципиальной или просто схемой. На ней все детали радиотехнического устройства и порядок их соединения изображают условными знаками, символизирующими эти детали, линиями. «Читая» принципиальную схему, как географическую карту или чертеж какого — то механизма, нетрудно разобраться в цепях и принципе работы устройства. Но она не дает представления о размерах устройства и размещении его деталей на монтажных платах. Схема соединений, в отличие от принципиальной, информирует, как расположены в конструкции и соединены между собой детали устройства. Собирая приемник, усилитель или любой другой радиоаппарат, радиолюбитель располагает детали и проводники примерно так, как на рекомендованной схеме соединений. Но монтаж и все соединения деталей проверяют по принципиальной схеме устройства. Уметь грамотно чертить и читать радиосхемы — совершенно обязательное условие для каждого, кто хочет стать радиолюбителем. На (рис. 6) вы видите уже знакомые вам детали и устройства и некоторые другие, с которыми придется иметь дело в дальнейшем. А рядом в кружках — их символические графические изображения на принципиальных схемах.

Рис. 6 Принципиальные схемы вариантов опытного приемника с настройкой переключением отводов катушки (а), конденсатором переменной емкости (б), ферритовым стержнем (в).

Любую катушку индуктивности без сердечника, независимо от ее конструкции и числа витков, на принципиальной схеме изображают в виде волнистой линии. Отводы катушек показывают черточками. Если катушка имеет неподвижный ферромагнитный сердечник (ферритовый стержень), увеличивающий ее индуктивность, его обозначают прямой линией вдоль изображения катушки. Если таким сердечником настраивают контур приемника, как это было в опытном приемнике, его на схеме обозначают то же прямой, но вместе с катушкой пересекают стрелкой. Подстроечный ферромагнитный сердечник катушки обозначают короткой жирной чертой, пересекающейся Т — образным символом. Любой конденсатор постоянной емкости изображают двумя короткими параллельными линиями, символизирующими две изолированные одна от другой пластины. Если конденсатор электролитический, его положительную обкладку обозначают дополнительным знаком « + ». Конденсаторы переменной емкости изображают так же, как и конденсаторы постоянной емкости, но пересеченными наискось стрелкой, символизирующей переменность емкости этого прибора. Гнезда для подключения провода антенны, головных телефонов или каких — то других устройств или деталей обозначают значками в виде вилки, а зажимы кружками. Новым для вас является переключатель. Вместо того, чтобы при настройке приемника раскручивать и скручивать проводники, как вы это делали во время опытов с детекторным приемником, выводы и отводы катушки можно переключать простейшим ползунковым, движковым или иной конструкции переключателем. Проводники, которыми соединяют детали, обозначают прямыми линиями. Если линии сходятся и в месте их пересечения стоит точка, значит проводники соединены. Отсутствие точки в месте пересечения проводников говорит о том, что они не соединены. На принципиальных схемах рядом с символическими обозначениями радиодеталей, приборов, коммутирующих и других устройств пишут соответствующие им латинские буквы. Так, например, всем конденсаторам, независимо от их конструктивных особенностей и применения, присвоена буква С, резисторам — буква R, катушкам — буква L, полупроводниковым диодам, транзисторам и многим другим полупроводниковым приборам — буква VD, V, антеннам буква W, гнездам и другим соединительным устройствам — буква X, головным телефонам, головкам громкоговорителей, микрофонам и другим преобразователям электрических или звуковых колебаний — буква В, гальваническим элементам и аккумуляторам — буква G, батареям гальванических элементов или аккумуляторов — буквы GB, лампам накаливания — буква Н и т.д. Кроме того, на схемах детали нумеруют, т. е. рядом с буквой, присвоенной детали, пишут цифру, например Cl, L1, L2, R1, VI и т.д. Для упрощения принципиальных схем на них иногда не показывают антенну, головные телефоны, ограничиваясь только обозначениями гнезд или зажимов для их подключения, но тогда возле них пишут соответствующие буквы с цифрами: Wl, В1. Подробнее об условном буквенно — цифровом позиционном обозначении радиотехнических элементов и устройств на схемах радиоаппаратуры можно найти в справочной литературе или интернете. Вот теперь, зная условные позиционные обозначения деталей, можно изобразить принципиальными схемами детекторные приемники, с которыми вы экспериментировали. Принципиальная схема первого варианта опытного приемника показана на (рис. 6, а). Его вы настраивали изменением числа секций катушки, входящих в контур, путем переключения заземлённого проводника. Поэтому в схему введен переключатель S1. Вспомните нашу «прогулку» по цепям приемника и совершите ее еще раз, но уже по принципиальной схеме. От начала катушки L1, обозначенного на схеме точкой, вы попадете к диоду VI и через него — телефонам В1, далее через телефоны по заземленному проводнику. Переключатель S1 и витки катушки Ll — к исходной точке. Это — детекторная цепь. Для токов высокой частоты путь из антенны в землю идет через секции катушки и переключатель. Это — антенный контур. Приемник настраивается на радиостанцию скачкообразным изменением числа витков, включаемых в контур. Параллельно телефонам подключен блокировочный конденсатор С1. На схеме штриховыми линиями показан конденсатор Са. В приемнике такой детали не было. Но символизирующая его электрическая емкость присутствовала — она образовывалась антенной и заземлением и как бы подключалась к настраиваемому контуру. Принципиальная схема одного из последующих вариантов опытного приемника показана на (рис. 6, б). Его входной настраиваемый контур состоит из катушки L1, имеющей один отвод, введенного вами конденсатора переменной емкости С2, антенного устройства и антенного конденсатора С1. Включение в контур только верхней (по схеме) секции катушки соответствует приему радиостанций СВ диапазона, включение обеих секций — приему радиостанций ДВ диапазона. Таким образом, в приемнике переход с одного диапазона на другой осуществляется переключателем S1, а плавная настройка в каждом диапазоне — конденсатором переменной емкости С2. Последним вариантом был приемник, настраиваемый ферритовым стержнем. Его принципиальную схему вы видите на (рис. 6, в). Колебательный контур образуют катушка L1 и конденсатор постоянной емкости С2. Катушка не имеет отводов, значит, приемник однодиапазонный. Для приема радиостанций другого диапазона в контур надо включить катушку, рассчитанную на прием станций этого диапазона. Для подключения головных телефонов предусмотрены гнезда В1.

Источник


Download Article


Download Article

Frequency, also called wave frequency, is a measurement of the total number of vibrations or oscillations made within a certain amount of time. There are a few different ways to calculate frequency based on the information you have available to you. Keep reading to learn some of the most common and useful versions.

  1. Image titled Calculate Frequency Step 1

    1

    Learn the formula. The formula for frequency, when given wavelength and the velocity of the wave, is written as: f = V / λ[1]

    • In this formula, f represents frequency, V represents the velocity of the wave, and λ represents the wavelength of the wave.
    • Example: A certain sound wave traveling in the air has a wavelength of 322 nm when the velocity of sound is 320 m/s. What is the frequency of this sound wave?
  2. Image titled Calculate Frequency Step 2

    2

    Convert the wavelength into meters, if necessary. If the wavelength is given in nanometers, you need to convert this value into meters by dividing it by the number of nanometers in a single meter.[2]

    • Note that when working with extremely small numbers or extremely large numbers, it is generally easier to write the values in scientific notation. The values will be shown in and out of their scientific notation forms for this example, but when writing your answer for homework, other schoolwork, or other formal forums, you should stick with scientific notation.
    • Example: λ = 322 nm
      • 322 nm x (1 m / 10^9 nm) = 3.22 x 10^-7 m = 0.000000322 m

    Advertisement

  3. Image titled Calculate Frequency Step 3

    3

    Divide the velocity by the wavelength. Divide the velocity of the wave, V, by the wavelength converted into meters, λ, in order to find the frequency, f.[3]

    • Example: f = V / λ = 320 / 0.000000322 = 993788819.88 = 9.94 x 10^8
  4. Image titled Calculate Frequency Step 4

    4

    Write your answer. After completing the previous step, you will have completed your calculation for the frequency of the wave. Write your answer in Hertz, Hz, which is the unit for frequency.

    • Example: The frequency of this wave is 9.94 x 10^8 Hz.
  5. Advertisement

  1. Image titled Calculate Frequency Step 5

    1

    Learn the formula. The formula for the frequency of a wave in a vacuum is almost identical to that of a wave not in a vacuum. Since there are no outside influences on the velocity of the wave, though, you would use the mathematical constant for the speed of light, which electromagnetic waves would travel at under these conditions. As such, the formula is written as: f = C / λ[4]

    • In this formula, f represents frequency, C represents the velocity or speed of light, and λ represents the wavelength of the wave.
    • Example: A particular wave of electromagnetic radiation has a wavelength of 573 nm when passing through a vacuum. What is the frequency of this electromagnetic wave?
  2. Image titled Calculate Frequency Step 6

    2

    Convert the wavelength into meters, if necessary. When the problem gives you the wavelength in meters, no further action is needed. If, however, the wavelength is given in micrometers, you need to convert this value into meters by dividing it by the number of micrometers in a single meter.

    • Note that when working with extremely small numbers or extremely large numbers, it is generally easier to write the values in scientific notation. The values will be shown in and out of their scientific notation forms for this example, but when writing your answer for homework, other schoolwork, or other formal forums, you should stick with scientific notation.
    • Example: λ = 573 nm
      • 573 nm x (1 m / 10^9 nm) = 5.73 x 10^-7 m = 0.000000573
  3. Image titled Calculate Frequency Step 7

    3

    Divide the speed of light by the wavelength. The speed of light is a constant, so even if the problem does not provide you with a value, the value remains 3.00 x 10^8 m/s. Divide this value by the wavelength converted into meters.[5]

    • Example: f = C / λ = 3.00 x 10^8 / 5.73 x 10^-7 = 5.24 x 10^14
  4. Image titled Calculate Frequency Step 8

    4

    Write your answer. With this, you should have calculated the value of the frequency of the wave. Write your answer in Hertz, Hz, the unit for frequency.

    • Example: The frequency of this wave is 5.24 x 10^14 Hz.
  5. Advertisement

  1. Image titled Calculate Frequency Step 9

    1

    Learn the formula. Frequency and the time taken to finish a single wave oscillation are inversely proportional. As such, the formula for calculating frequency when given the time taken to complete a wave cycle is written as: f = 1 / T

    • In this formula, f represents frequency and T represents the time period or amount of time required to complete a single wave oscillation.
    • Example A: The time for a certain wave to complete a single oscillation is 0.32 seconds. What is the frequency of this wave?
    • Example B: In 0.57 seconds, a certain wave can complete 15 oscillations. What is the frequency of this wave?
  2. Image titled Calculate Frequency Step 10

    2

    Divide the number of oscillations by the time period. Usually, you will be told how long it takes to complete a single oscillation, in which case, you would just divide the number 1 by the time period, T. If given a time period for numerous oscillations, however, you will need to divide the number of oscillations by the overall time period required to complete them.[6]

    • Example A: f = 1 / T = 1 / 0.32 = 3.125
    • Example B: f = 1 / T = 15 / 0.57 = 26.316
  3. Image titled Calculate Frequency Step 11

    3

    Write your answer. This calculation should tell you the frequency of the wave. Write your answer in Hertz, Hz, the unit for frequency.

    • Example A: The frequency of this wave is 3.125 Hz.
    • Example B: The frequency of this wave is 26.316 Hz.
  4. Advertisement

  1. Image titled Calculate Frequency Step 12

    1

    Learn the formula. When told the angular frequency of a wave but not the standard frequency of that same wave, the formula to calculate the standard frequency is written as: f = ω / (2π)[7]

    • In this formula, f represents the frequency of the wave and ω represents the angular frequency. As with any mathematical problem, π stands for pi, a mathematical constant.
    • Example: A particular wave rotates with an angular frequency of 7.17 radians per second. What is the frequency of that wave?
  2. Image titled Calculate Frequency Step 13

    2

    Multiply pi by two. In order to find the denominator of the equation, you need to double the value of pi, 3.14.

    • Example: 2 * π = 2 * 3.14 = 6.28
  3. Image titled Calculate Frequency Step 14

    3

    Divide the angular frequency by the double of pi. Divide the angular frequency of the wave, given in radians per second, by 6.28, the doubled value of pi.[8]

    • Example: f = ω / (2π) = 7.17 / (2 * 3.14) = 7.17 / 6.28 = 1.14
  4. Image titled Calculate Frequency Step 15

    4

    Write your answer. This final bit of calculation should indicate what the frequency of the wave is. Write your answer in Hertz, Hz, the unit for frequency.

    • Example: The frequency of this wave is 1.14 Hz.
  5. Advertisement

Add New Question

  • Question

    What is the frequency if 80 oscillations are completed in 1 second?

    Community Answer

    Frequency is the number of oscillations completed in a second. The answer would be 80 Hertz.

  • Question

    Do atoms have a frequency and, if so, does it mean everything vibrates?

    Donagan

    Atoms have energy. Energy is often characterized as vibration. Vibration possesses frequency. So, yes, everything could be thought of as vibrating at the atomic level.

  • Question

    What’s the definition of frequency?

    Community Answer

    The rate at which a vibration occurs that constitutes a wave, either in a material (as in sound waves), or in an electromagnetic field (as in radio waves and light), usually measured per second. The rate at which something occurs or is repeated over a particular period of time or in a given sample.

See more answers

Ask a Question

200 characters left

Include your email address to get a message when this question is answered.

Submit

Advertisement

Thanks for submitting a tip for review!

Things You’ll Need

  • Calculator
  • Pencil
  • Paper

References

About This Article

Article SummaryX

To calculate the frequency of a wave, divide the velocity of the wave by the wavelength. Write your answer in Hertz, or Hz, which is the unit for frequency. If you need to calculate the frequency from the time it takes to complete a wave cycle, or T, the frequency will be the inverse of the time, or 1 divided by T. Display this answer in Hertz as well. Keep reading to learn how to calculate frequency from angular frequency!

Did this summary help you?

Thanks to all authors for creating a page that has been read 1,511,957 times.

Did this article help you?

Raizor

Новобранец

    • Поделиться

Здравствуйте, имеется передатчик 2-х канальный, аля брелок автосигнализации. хотелось бы узнать частоту его сигналов и собрать приемник для дистанционного запуска компа (1-го канала хватит).

  • Цитата

Ссылка на комментарий
Поделиться на другие сайты

STEN50

Мастер

    • Поделиться

частотомер или осциллограф.

  • Цитата

Прогноз прохождения всегда точен.Бывает ,что не совпадают  место,время и

 частота…73!Roman ex Call:RA3WSI.  6П3С-Лампа легенда!!!

Страсть как люблю когда *спасибку* нажимают ;)

Ссылка на комментарий
Поделиться на другие сайты

Raizor

Новобранец

  • Автор
    • Поделиться

осциллографа нет( да и частотометра тоже(

  • Цитата

Ссылка на комментарий
Поделиться на другие сайты

ИБП MEAN WELL серии DRC-180 на DIN-рейку – новое решение для пожарно-охранных систем

Компания MEAN WELL расширила семейство DRC-40/60/100 – недорогих ИБП (UPS) 2-в-1 (ИП и контроллер заряда/разряда АКБ в одном корпусе) с креплением на DIN-рейку. Теперь доступны модели мощностью 180 Вт новой серии DRC-180.

Источник питания DRC-180 предназначен для создания систем бесперебойного питания с внешней АКБ и может использоваться в охранно-пожарных системах, системах аварийной сигнализации, контроля доступа и в других приложениях, где требуется простая, недорогая и в то же время качественная система бесперебойного питания соответствующей мощности. Подробнее>>

STEN50

Мастер

    • Поделиться

Тогда увы.

  • Цитата

Прогноз прохождения всегда точен.Бывает ,что не совпадают  место,время и

 частота…73!Roman ex Call:RA3WSI.  6П3С-Лампа легенда!!!

Страсть как люблю когда *спасибку* нажимают ;)

Ссылка на комментарий
Поделиться на другие сайты

25.05.2023 Вебинар «Источники питания MORNSUN: новинки для промавтоматики и оптимальные решения для телекоммуникации»
Приглашаем на вебинар, посвященный новой продукции MORNSUN для промышленной автоматизации и телекоммуникационных приложений.
Мы представим источники питания на DIN-рейку класса High-End для применения в ответственных областях, способные заменить продукцию именитых европейских брендов, а также безвентиляторные ИП для жестких условий эксплуатации, модули UPS и резервирования. Рассмотрим, как и на базе каких компонентов можно реализовать питание в телекоммуникационных и промышленных устройствах от шины до точки нагрузки (PoL). Покажем, почему использование продукции MORNSUN выгодно в нынешних экономических условиях.
Подробнее>>

минздрав

Мастер

    • Поделиться

Тогда прямо никак, только забить в поиск название и рыть до…

  • Цитата

Хорошо зафиксированный пациент в анестезии не нуждается

Ссылка на комментарий
Поделиться на другие сайты

Raizor

Новобранец

  • Автор
    • Поделиться

блин( очень обидно.

а может подскажите, хотя бы примерно, схемку приемника для моей цели?

  • Цитата

Ссылка на комментарий
Поделиться на другие сайты

Igel

Мастер

    • Поделиться

афигенно… я так понимаю что раз частоту передатчика узнать не получилось то и приёмник всё равно на какую частоту нужен? :)

  • Цитата

Ссылка на комментарий
Поделиться на другие сайты

Лалюна

Опытный

    • Поделиться

Можно попробовать пытать паяльником пока не сознается…

Или найти на плате кварц или резонатор ПАВ(если есть конечно)и почитать чего на ём пишут.

  • Цитата

Не квалифицированный, но везучий электрик знает в лицо всех архангелов.

Прогресс сделал розетки недоступными для большинства детей. Умирают только самые одаренные.

Ссылка на комментарий
Поделиться на другие сайты

Raizor

Новобранец

  • Автор
    • Поделиться

Нет, с приемником на конкретную частоту разберемся, когда её узнаем. я хочу узнать узнать принцип работы после получения сигнала, мне нужно кратковременное замыкание 2-х подведенных контактов (вместо кнопки).

Лалюна, понял, буду разбирать)

Ребята, извиняйте, ступил конкретноlaugh.gif на бумажке в комплекте написана частота – 433,92 MHz. теперь всё же нужно собрать для него приемник)

  • Цитата

Ссылка на комментарий
Поделиться на другие сайты

Igel

Мастер

    • Поделиться

ну, не хотел вас расстраивать раньше времени но придётся…

433,92 – стандартная частота автомобильных сигналок, если вы сделаете приёмник просто на превышение порогового уровня то любой брелок в радиусе 30 метров будет вызывать срабатывание, а что б декодировать посылку надо процессор, а это КМК вам будет не по силам, да там ещё и КиЛог скорее всего…

  • Цитата

Ссылка на комментарий
Поделиться на другие сайты

Raizor

Новобранец

  • Автор
    • Поделиться

хм..а если сигнал немного приглушать уже на приемнике? ну т.е. чтобы ловило максимум в 2-3-х метрах?

у меня частный дом, не так много машин с сигнализацией вокруг..

  • Цитата

Ссылка на комментарий
Поделиться на другие сайты

Igel

Мастер

    • Поделиться

ну так что, всё равно любой брелок или любой клоун с портативной станцией… если просто лампочку зажигать – это одно, ежли запирать чего – совсем другое

  • Цитата

Ссылка на комментарий
Поделиться на другие сайты

Raizor

Новобранец

  • Автор
    • Поделиться

так если радиус действия будет 2-3 метра, то другой брелок просто не достанет, клоунов со станциями не наблюдалось…а использовать хочу как кнопку включения компьютера…

  • Цитата

Ссылка на комментарий
Поделиться на другие сайты

Igel

Мастер

ulura

Специалист

    • Поделиться

А ко мне как то обратился товарищ, который оставил машину под телевышкой. Закрыть – закрыл. А открыть не смог…Смешно не было. Машина дорогая и, самое главное – чужая. Будете смеяться,но открыли с помощью пищевой фольги. Размотали метра 3, укрыли хозяина с брелоком и с машиной. Сработало. А чтобы оценить ( а может и измерить) частоту передачи брелка – можно использовать анализатор спектра.


Изменено 13 августа, 2012 пользователем ulura

  • Цитата

“Грехи других судить Вы так усердно рветесь, – начните со своих, и до чужих не доберетесь”

Уильям Шекспир

Ссылка на комментарий
Поделиться на другие сайты

Raizor

Новобранец

  • Автор
    • Поделиться

спасибо, буду осваивать)

  • Цитата

Ссылка на комментарий
Поделиться на другие сайты

SDD39

Знаток

  • 1 месяц спустя…

android0350

Знаток

    • Поделиться

Сделал жучка, осталось узнать частоту на которой он работает. Нужен недорогой прибор чтобы её узнать (самодельный не вариант). Осциллографы дорогие, да и функций там полно ненужных для меня.

  • Цитата

Электроника^_^

Ссылка на комментарий
Поделиться на другие сайты

salyamkamrad

Профи

    • Поделиться

Откуда такая уверенность, что жучок работает? А частота измеряется частотомером. В интернет-магазинах их навалом всяких, от простых конструкторов до сложнейших сверхточных приборов.

  • Цитата

Ссылка на комментарий
Поделиться на другие сайты

android0350

Знаток

    • Поделиться

Частотомер конструктор, это самодельный что ли?

  • Цитата

Электроника^_^

Ссылка на комментарий
Поделиться на другие сайты

минздрав

Мастер

    • Поделиться

Сделал жучка, осталось узнать частоту на которой он работает.

Это такая шутка?

Выглядит так: сделал машинку, осталось узнать – нахрена.

  • Цитата

Хорошо зафиксированный пациент в анестезии не нуждается

Ссылка на комментарий
Поделиться на другие сайты

РадиоКот >Обучалка >Аналоговая техника >Жучки, передатчики и приемники: что о них надо знать >

Основные параметры передатчиков и приемников

Основные параметры передатчиков

Основные параметры приемников

Рабочая частота (частотный диапазон), МГц или кГц

Тип модуляции: амплитудная (АМ) / частотная (ЧМ)

Мощность выходного сигнала, Вт

Чувствительность по входу, мкВ

Выходное сопротивление, Ом

Входное сопротивление, Ом

Коэффициент гармоник

Избирательность, дБ

Чувствительность по входу, мВ

Мощность выходного сигнала, Вт

Коэффициент нелинейных искажений (КНИ) НЧ тракта (включая модулятор)

КНИ НЧ тракта, включая демодулятор

Теперь все по порядку.

Рабочая частота (частотный диапазон)

Если передатчик или приемник жестко настроены на определенную частоту – то можно говорить об одной рабочей частоте. Если в процессе работы возможно перестраивать рабочую частоту, то надо назвать диапазон рабочих частот, в пределах которого может осуществляться регулировка.

Измеряется в килогерцах (кГц), мегагерцах (МГц) или гигагерцах (ГГц).

Раньше для определения частотного диапазона чаще использовали не частоту, а длину волны. Отсюда пошли названия диапазонов ДВ (длинные волны), СВ, (средние волны) КВ (короткие волны), УКВ (ультракороткие волны).

Чтобы пересчитать длину волны в частоту, нужно поделить на нее скорость света (300 000 000 м/с). То есть,

где:
– длина волны (м)

c – скорость света (м/с)

F – частота (Гц)

Теперь вам нетрудно посчитать, что наши деды называли «ультракороткими волнами». Да да, не удивляйтесь, диапазон 65…75 МГц – это уже не просто «короткие» а «ультракороткие». А ведь их длина целых 4 метра! Для сравнения, длина волны мобильника стандарта GSM – 15…30 см (в зависимости от диапазона).

С развитием техники и освоением новых частотных диапазонов, им начали давать невообразимые названия вроде «сверхкороткие», «гиперкороткие» и т.п. Сейчас для обозначения диапазона чаще используют частоту. Это удобнее хотя бы даже тем, что не нужно ничего пересчитывать и помнить скорость света. Хотя, скорость света все равно помнить не помешает 🙂

Мы будем, в основном, работать с вещательными диапазонами УКВ. Их два: УКВ-1 – то что в народе так и называют “УКВ”, и УКВ-2 – то, что принято называть “FM”. Название FM происходит от английского Frequency Modulation – Частотная Модуляция (о модуляции читаем ниже). Вообще-то, если серьезно, то называть частотный диапазон по виду модуляции – технически безграмотно. Однако, в народе это название прочно укоренилось и стало нарицательным. С этим уже ничего не поделаешь.

Тип модуляции

Широко используется два типа модуляции: амплитудная (АМ) и частотная (ЧМ). По-буржуйски это звучит как AM и FM . Собственно, всеми любимый диапазон “ FM ” получил название именно благодаря частотной модуляции, с которой работают все радиостанции данного диапазона. Есть еще фазовая модуляция, сокращенно – ФМ, но уже, нашенскими буквами. Попрошу не путаться с буржуйским FM !

ЧМ, в отличие от АМ, более защищена от импульсных помех. Вообще говоря, на частотах, на которых расположены радиостанции УКВ-диапазона, применение ЧМ более удобно, чем АМ, поэтому она там и применяется. Хотя, телевизионный сигнал все равно передается с амплитудной модуляцией, независимо от частоты. Но это уже совсем другая история.

Частотная модуляция бывает узкополосная и широкополосная. В вещательных радиостанциях используется широкополосная ЧМ – ее девиация составляет 75 кГц.

В связных радиостанциях и прочей не вещательной радиотехнике чаще применяют узкополосную ЧМ, с девиацией порядка 3 кГц. Она более защищена от помех, поскольку допускает более острую настройку приемника на несущую.

Итак, наши диапазоны:

УКВ-1 – 65,0…74,0 МГц, модуляция – частотная

УКВ-2 (“FM”) – 88,0…108,0 МГц, модуляция – частотная

Мощность выходного сигнала

Чем мощнее передатчик – тем дальше он может передать сигнал, тем легче этот сигнал будет принять.

Почти в каждом описании жучка пишется его дальность действия. Обычно – начиная от 50 м и заканчивая тремя километрами… Серьезно воспринимать эту информацию нельзя. Ни за что не польщайтесь дальностью в 1 км в условиях города, или не расстраивайтесь сильно пятидесятью метрами на открытой местности – ведь авторы никогда не дают параметры приемника, с которым тестировался данный жучок. А именно – они не называют чувствительность этого приемника. А ведь от нее многое зависит. Можно тестировать мощный передатчик при помощи приемника с паршивой чувствительностью – и получить в результате маленький радиус действия. Или наоборот, слушать маломощный передатчик через чувствительный приемник – и получить большую дальность. Поэтому, рассматривая схему жучка, в первую очередь обращайте внимание не на громкие слова, а на голые факты. А именно – попытайтесь прикинуть мощность передатчика. Обычно мощность в описании жучка не указывается (авторы ее просто не меряют, считая достаточным померить «дальность»). Поэтому нам остается только «на глаз» определить, на что способен жук.

Для этого нужно смотреть на:

– Напряжение питания. Чем больше – тем больше мощность (при прочих равных условиях)

– Номинал транзистора, стоящего в оконечном каскаде (или генераторе, если антенна подключена прямо к нему). Если стоит какой-нибудь паршивый КТ315 – большой мощности от схемы можно не ждать, не дождетесь. А если попробуете поднять – транзюк, ничего не говоря, просто предательски взорвется… Лучше, если стоит транзистор КТ6хх или КТ9хх, например, КТ608, КТ645, КТ904, КТ920 и т.д.

– Сопротивления транзисторов в коллекторной и эмиттерной цепях оконечного каскада. Чем они меньше – тем больше мощность (ппру).

Для сравнения скажу так: мощности в 1 Вт хватает в городских условиях где-то на километр при условии, что чувствительность приемника – порядка 1мкВ.

Чувствительность приемника

Ну мы уже начали говорить о чувствительности.

Чувствительность зависит процентов на 90 от «шумности» входного каскада приемника. Поэтому, для достижения хороших результатов, необходимо использовать малошумящие транзисторы. Часто используют полевики – они поменьше шумят.

У приемников диапазона УКВ, чувствительность обычно находится в пределах 0,1…10мкВ. Приведенные значения – крайности. Чтоб получить чувствительность 0,1 – надо изрядно попотеть. Так же, как и надо очень сильно не уважать себя, чтоб сделать приемник с чувствительностью 10мкВ. Истина где-то посередине. Порядка 1…3 мкВ – оптимальное значение чувствительности.

Выходное сопротивление передатчика

Это очень важно знать, потому что можно сделать очень прекрасный мощный передатчик и не получить от него и десятой доли номинальной мощности благодаря неправильному согласованию с антенной.

Итак, антенна обладает сопротивлением R , скажем 100 Ом. Чтоб излучить при помощи этой антенны мощность P , допустим – 4 Ватта, нужно приложить к ней напряжение U , которое рассчитывается по закону Ома:

U2 = PR
U2 = 100*4 = 400
U = 20 В

Получили 20 Вольт.
При напряжении 20 Вольт выходной каскад передатчика должен держать мощность 4 Вт, при этом через него будет протекать ток

I = P/U = 0,2А = 200мА

Таким образом, данный передатчик на сопротивлении 100 Ом развивает мощность 4 Вт.
А если вместо антенны на 100 Ом подключить антенну на 200 Ом? (А напряжение то же – 20 В)

Считаем:
P = UI = U(U/R) = 20(20/200) = 2 Вт

В два раза меньше! То есть, физически, выходной каскад готов прокачать 4 Ватта,
но не может, так как ограничен напряжением в 20 Вольт.

Другая ситуация: сопротивление антенны – 50 Ом, то есть – в 2 раза меньше. Что получается? На нее пойдет двойная мощность, через оконечный каскад потечет двойной ток – и транзистор в конечном каскаде многозначительно накроется медным тазом…

Короче говоря, к чему я это все? А к тому, что необходимо знать, какую нагрузку мы вправе подключить к выходу передатчика, а какую – не в праве. То есть, необходимо знать выходное сопротивление передатчика.

Но нам надо знать и сопротивление антенны. А вот тут-то сложнее: его очень сложно измерить. Можно, конечно, рассчитать, но расчет не даст точного значения. Теория всегда немного расходится с практикой. Как же быть?

Очень просто. Существуют специальные схемы, которые позволяют изменять выходное сопротивление. Они называются «схемы согласования». Наиболее распространены два вида: на основе трансформатора и на основе П-фильтра. Схемы согласования обычно ставятся на выходной каскад усилителя, и выглядят примерно так (слева – трансформаторная, справа – на основе П-фильтра):

Схемы согласования сопротивлений

Для настройки выходного сопротивления трансформаторной схемы, необходимо изменять количество витков II обмотки.

Для настройки схемы с П-фильтром, нужно регулировать индуктивность L 1 и емкость C 3.

Настройка производится при включенном передатчике и подключенной штатной антенне. При этом, мощность излученного антенной сигнала измеряется при помощи специального прибора – волномера (это такой приемничек с милливольтметром). В процессе настройки, добиваются максимального значения излучаемой мощности. Крайне не рекомендуется производить настройку мощных передатчиков, находясь в непосредственной близости от антенны. Если, конечно, ваша мама хочет иметь внуков… 🙂

Входное сопротивление приемника

Почти то же самое. Кроме внуков. Принимаемый сигнал слишком слаб, чтобы сколь-нибудь навредить отечественному генофонду.

Согласование сопротивлений производится при помощи входного колебательного контура. Антенна подключается либо к части витков контура, либо через катушку связи, либо через конденсатор. Схемы вот:

Входные схемы приемников

Сигнал с контура также может сниматься или напрямую, как показано на схемах, или через катушку связи, или с части витков. Во-общем, зависит от воли конструктора и конкретных условий.

Коэффициент гармоник

Говорит нам о том, насколько излучаемый передатчиком сигнал «синусоидален». Чем меньше к.г. – тем больше сигнал похож на синус. Хотя, бывает и так, что визуально – вроде бы синус, а гармоник – тьма. Значит, все-таки – не синус. Человеку свойственно ошибаться. Техника более объективна в своей оценке.

Вот так выглядит «чистый» синус (синусоида сгенерирована звуковым генератором программы WaveLab ):

Чистый палэзный и нэдарагой синус

Гармоники возникают, как мы знаем, из-за нелинейных искажений сигнала. Искажения могут возникать по различным причинам. Например, если усилительный транзистор работает на нелинейном участке передаточной характеристики. Иначе говоря, если при равных изменениях тока базы, изменения тока коллектора не равны. Это может быть в двух случаях:

  1. На транзистор подан недостаточный ток смещения. То есть, при отсутствии сигнала он полностью закрыт,
    а открываться начинает лишь с возрастанием уровня сигнала. При этом, у выходной синусоиды получаются «спиленными» низы:

    Синус со срезанными низами

    Кстати, выходные каскады большинства передатчиков работают в режиме С.
    Этот режим не подразумевает наличия смещения базы. То есть, на выходах таких каскадов
    всегда будет сигнал с отрезанными низами. Но с этим мирятся ввиду высокого КПД подобных каскадов.
    Гармоники вырезаются фильтрами, стоящими после каскада. Кстати, каскады, изображенные на схемах согласования,
    работают как раз в режиме C.

  2. Амплитуда входного сигнала слишком велика, и необходимый коллекторный ток не может быть обеспечен.

    Например:
    В коллекторной цепи транзистора стоит резистор на 100 Ом,
    напряжение питания – 25 В.
    Соответственно, при полностью открытом транзисторе, коллекторный ток будет равен 25/100 = 0,25 А = 250 мА.
    Коэффициент усиления транзистора– 50, то есть, коллекторный ток в 50 раз больше тока базы.
    Теперь такая ситуация: на базу подали ток 10 мА. Каков будет ток коллектора?

    Что? 500 мА? Ни фига подобного! Мы же только что говорили, что при ПОЛНОСТЬЮ открытом транзисторе, коллекторный ток составляет 250 мА. Значит, больше этого значения, он не сможет быть ни под каким соусом. Если мы будем увеличивать ток базы от нуля до 10 мА, то коллекторный ток будет возрастать только до тех пор, пока не станет равным 250 мА. После этого, он не увеличится, сколько бы мы не увеличивали ток базы. Такой режим транзистора называется « режим насыщения ». В момент достижения коллекторным током отметки 250 мА, базовый ток равен 250/50 – 5 мА. То есть, для корректной работы данного каскада, на его вход нельзя подавать ток больше 5 мА.

    То же самое происхедит и с сигналом. Если ток сигнала «зашкаливает» за определенное значение, то транзистор уходит в насыщение. На осциллограмме это проявляется в виде «спиленных» верхушек синусоиды:

  3. Синус со срезанными верхушками

Кроме таких характерных искажений, возникают и другие всевозможные нелинейные искажения сигнала. Со всеми этими искажениями призваны бороться частотные фильтры. Обычно, используются фильтры нижних частот (ФНЧ), поскольку, как говорилось ранее, частоты гармоник обычно выше частоты полезного сигнала. ФНЧ пропускает основную частоту и «вырезает» все частоты, которые выше основной. При этом, сигнал, как по волшебству, превращается в синус чистой красоты.

Избирательность приемника.

Этот параметр показывает, насколько хорошо приемник может отделить сигнал требуемой частоты от сигналов других частот. Измеряется в децибелах (дБ) относительно соседнего частотного канала либо зеркального канала (в гетеродинных приемниках).

Дело в том, что в эфире постоянно летят тысячи всевозможных электромагнитных колебаний: от радиостанций, телевизионных передатчиков, наших любимых «мобильных друзей», и т.д. и т.п. Различаются они лишь по мощности да по частоте. Правда, по мощности им отличаться не обязательно – это не есть критерий выбора. Настройка на любую радиостанцию, будь то телеканал « MTV » или база вашего домашнего радиотелефона, происходит именно по частоте. При этом, на приемнике лежит ответственность: выбрать из тысяч частот – ту одну, единственную и неповторимую, которую мы хотим принять. Если на близких частотах нет никаких признаков разумной жизни – хорошо. А если где-нибудь через пол-мегагерца от нашей радиостанции, находится сигнал другой радиостанции? Это есть не очень хорошо. Вот тут то и понадобится хорошая избирательность приемника.

Избирательность приемника зависит, в-основном, от добротности колебательных контуров. Подробнее, мы будем разбираться с избирательностью при рассмотрении конкретных схем приемников.

Оставшиеся четыре параметра относятся к НЧ тракту приемника и передатчика.

Чувствительность по НЧ входу передатчика

Чем чувствительнее вход передатчика, тем более слабый сигнал можно на него подавать. Этот параметр особенно важен в жучках, где сигнал снимается с микрофона, и имеет очень малую мощность. Если нужно, чувствительность наращивается дополнительными каскадами усиления.

Мощность выходного НЧ-сигнала приемника

Мощность сигнала, которую отдает на выход приемник. Ее необходимо знать, чтобы правильно подобрать усилитель мощности для дальнейшего усиления.

КНИ (Коэффициент нелинейных искажений)

Ну, в-общем, мы уже разобрались, что такое нелинейные искажения и откуда они берутся. Но! Если по ВЧ-тракту достоточно поставить фильтр – и все станет хорошо, то в звуковом тракте «лечить» нелинейные искажения куда труднее. Точнее – просто невозможно. Поэтому, со звуковым или любым другим модулирующим сигналом, необходимо обращаться очень бережно, чтобы в нем возникло как можно меньше нелинейных искажений.

<<–Вспомним пройденное—-Поехали дальше–>>


Как вам эта статья?

Заработало ли это устройство у вас?

Добавить комментарий